Download as pdf
Download as pdf
You are on page 1of 159
THE CONTEST PROBLEM BOOK PROBLEMS FROM THE ANNUAL HIGH SCHOOL CONTESTS OF THE MATHEMATICAL ASSOCIATION OF AMERICA Compiled and with solutions by CHARLES T. SALKIND A nickel is placed on a table. What is the number of nickels which can be placed around it so that each is tangent to it and to two others? New Mathematical Library THE CONTEST PROBLEM BOOK Problems from the Annual High School Contests of the Mathematical Association of America compiled and with solutions by Charles T. Salkind Brooklyn Polytechnic Institute RANDOM HOUSE lustrations by Florence W. Cochrane First Printing © Copyright, 1961, by Yale Univesity Alright reserved under International and Pan-American Copyright Convention ished in New York by Random House, Ine, and simultaneously in Toronto, Canada, by Random House of Canada, Limited. Library of Congress Catalog Card Number: 61-13843 Manufactured in the United States of America mm Vv CONTENTS Preface Problems Answer Keys Solutions Classification of Problems 75 9 150 PREFACE Mathematical problems are older than mathematics itself. Stated as puzzles, they are found in the oldest written records. An Egyptian Papyrus, dating back to 2200 n.c., asks: “A heap and its seventh make 19, how large is the heap?” Although this is stated almost as tersely as we would do it today (“if 2 + (1/T)z = 19, find z”), ancient prob- lems often appear in more poetical garb. Here is a charming example from Indie (recorded about 1150 4.0.): Out of a heap of pure lotus flowers, a third part, a fifth and a sixth were offered respectively to the gods Siva, Vishnu, and then Sun; a quarter of the original heap was presented to Bhavani. The remaining six lotuses were given to the venerable preceptor. Tell quickly the whole number of lotus flowers. This sounds more exciting than: Solve the equation eit tied 6-5 but of course our way of stating the problem has an overriding ad- vantage: it makes the solution easier. The glory of elevating mathematics from the level of problem solv- ing to that of a science belongs to the ancient Greeks. But these Greek mathematicians did not neglect the art of inventing and solving prob- lems. Three of their proposed geometric problems remained challenges to mathematicians until the 19th century. The “three famous prob- lems of antiquity” asked for the trisection of an angle, the doubling of s cube, and the squaring of a circle, using only straight edge and compass. In spite of centuries of failure with these problems, efforts 1 2 THE CONTEST PROBLEM BOOK continued until it was proved, with the aid of algebraie theorems, that, in each case, the construction is impossible with the specified tools. Since the time of the Greeks, such problems have stimulated the growth of mathematics and led mathematicians to invent new methods tnd to develop new concepts. On # more modest but equally important level, such problems have provided an unexcelled training ground for young mathematicians. But problem-solving never was the exclusive domain of professionals. At all times it provided intellectual stimula- tion and joy for many professionally outside the field of mathematics. During the Renaissance in Italy problem-solving became a compet tive sport and publie contests between mathematicians were not un- common. Sometimes the competitive spirit led to excesses. For example, in the 16th century, Tartaglia, the winner of such a contest, had to flee town to escape violence at the hands of the fans of the local cham- pion. The subject of this particular contest was cubic equations and, in preparing for it, Tartaglia discovered important formulas for solving them. In our own day mathematicians still compete with each other in solving problems—less publicly, less violently, but perhaps with the same intensity. The end of the 19th century saw the beginning of organized com- petitions among secondary school students. The so-called Edtvos Competitiont in Hungary (begun in 1804) is justly famous; it has probably played its part in producing so many superior mathemati- cians and physicists from this small country. In the Soviet. Union high school students take part in » university-sponsored system of mathematical “olympics” In America the tradition of periodic competitions among students, of mathematics is almost fifty years old. There are, for example, the Inter-scholastic Mathematics League in New York City and Long Island and the annual competitions conducted by Pi Mu Epsilon of New York University; there are also state-wide and regional programs in Texas, Wisconsin, Indiana, and at Stanford University, and else- where, As our first problem book we present: the complete set of prob- lems proposed by the Mathematical Association of America in its ‘annual contests for high school students. {Translations of these problems and solutions will appear in the NMC. serie. PREFACE . ‘The MAA is concerned primarily with mathematics on the under- graduate level. It is one of the three major mathematical organisa tions in America (the other two being the American Mathematical Society, chiefly concemed with mathematical research, and the Na- tional Council of Teachers of Mathematics, concerned with the content and pedagogy of elementary and secondary mathematics). The MAA also conducts the annual “Putnam Competition” for under-graduate students. Its journal, The American Mathematical Monthly, is famous for its elementary and advanced problem sections. ‘When the MAA contest was first organized in 1960 it was restricted to the Metropolitan New York area. It became national project in 1957, receiving in that same year co-sponsorship by the Society of Actuaries. In 1960 more than 150,000 students from 5,200 schools participated in the program. The contest is conducted in nearly every state and territory of the U. 8. and in the more-populated provinces of Canada. ‘The MAA contest is based entirely on the standard high school curriculum (except for » very few problems in the earlier tests) and ppre-supposes no advanced knowledge. Readers who find these problems too easy are advised to try the problem sections of other NML books and to await additional NML problem books which are now in prepara- tion, Part I of each examination tests fundamental skills based on con- ceptual understanding, while Parts II and III probe beyond mere reproduction of class-room work. For the ten years this contest was conducted, only three people scored perfectly (150); a score of 80 oF more places a contestant on the Honor Roll. ‘The editors are grateful to the MAA for permission to publish this collection, and to Prof. Charles T. Salkind for compiling the book and for supplying a classification of problems with their complete solutions In preparing this collection, the compiler and the editors have made 1 few minor changes in the statements of the original contest problems, for the sake of greater clarity, { Scoring iss follows: 1950-1989 Part 1 2 points each, Part If 8 points exch, Part 1114 points each; 1960 Part 8 points each, Part IT 4 points each, Part 111 5 points each, SUGGESTIONS FOR USING THIS BOOK This problem collection is designed to be used by mathematics d-10 ce Acar travels 120 miles ftom A to Bat 30 miles per hour but returns the same distance at 40 miles per hour. The average speed for the round trip i closest to (A) 38 mph(B) 4 mph (C) 85 mph (D) 36 mph CE) 37 mph 8 THE CONTEST PROBLEM BOOK 28, Two boys A and B start at the same time to ride from Port Jervis to Poughkeepsie, 60 miles away: A travels 4 miles an hour slower than B. B reaches Poughkeepsie and at once turns back meeting A 12 miles from Poughkeepsie. The rate of A was: (A)4mph—(B)8 mph (C) 12 mph (D) 16 mph (E) 20 mph 2. A maiiufacturer built « machine which will address 500 envelopes in 8 ‘minutes. He wishes to build another machine so that when both are ‘operating together they will addrese 500 envelopes in 2 minutes. The equation used to find how many minutes x it would require the second ‘machine to addres 500 envelopes alone - eerie 600 ol (As—2=2 (BE+l=5 (c+ P- 500 (D)3+Z= 1 (B) none of these answers 30, From a group of boys and girls, 15 girls leave. There are then left two boys for each girl. After this 45 boys leave. There are then 5 girls for each boy. The number of girls in the beginning was: (A) 40 (B) 43. (C)29 (D) 50 (E) none of these 31. John ordered 4 pairs of black socks and some additional pairs of blue socks. The price of the black socks per pair was twice that of the blue. ‘When the order was filled, it was found that the number of pairs of the ‘two colors had been interchanged. This increased the bill by 50%. The ratio of the number of pairs of black socks to the number of pairs of blue socks in the original order was: (A) 4:1 (B)21 (Ce (12 Bus 82. A.25 foot ladder is placed against a vertical wall of a building. The foot of the ladder is 7 feet from the base of the building. If the top of the ladder slips 4 feet, then the foot of the ladder will side: At Bot Ost Met Hast. 88. The number of circular pipes with an inside diameter of 1 inch which will carry the same amount of water as a pipe with an inside diameter of 6 inches is: (A) Gr (B)G (C)12_ (D) 36 (E) 36 34, When the circumference of a toy balloon is increased from 20 inches to 25 inches, the radius is increased b) “ (B)2hin. (C)5/xin. (D) 5/2rin. (B) ¥/5 35. In AABC, AC = 4", BC = 10", and AB = 26". The radius of the inseribed circle is (A) in, (B) din. (©) 13: (D) Bin. (B) none of these 36. 37 41 PROBLEMS: 1950 EXAMINATION ° Part 3 ‘A merchant buys goods at 25% off the list price. He desires to mark the ‘Goods #0 that he can give « discount of 20% on the marked price and still clear a profit of 25% on the selling price. What per cent of the list price muat he mark the goods? (A) 125% —(B) 100% (C) 120% (D) 80% (B) 75% If y= loger, and a > 1, which of the following statements is incorrect? Wit z=1,y (Bit z=, y=1 (©) if z= 1, y is imaginary (complex) (D) if 0 <2 <1, y is always less than 0 and decreases without limit as 2 approaches sero (B) only some of the above statements are correct Ur the exprenion [9 §| has thevalue ob — ed forall valusot o, by ¢ and 4, then the equation [22] = 3: (A) is satisfied for only 1 value of = (B) is satisfied for 2 values of x (© is atin for no vals of = (D) is satiafied for an infinite number of values of x (E) none of these Given theseres 2+14)4 $4 --- and thefolowing Gvestatements: (0) the sum increases without iis. (@) the sum decreases without limit GG) the diference between any term of the sequence and zero can be made lea than any positive quantity no matter how eral (4) the difference between th sum and 4 can be made les than any posi tive quantity no matter how smal (6) the sum approaches a limit. Of those statements, the correct ons are (A) only (8) and (4) (B) only (8) (©) only (2) and (4) @) only @), @) and (4) CB) only (4) and () The limit of 2 as = approaches 1 asa limit, (A) 0 (B) indeterminate (C)z—1 (D)2 1 ‘The least value of the function az* + bz + ¢(a > 0) is: b b 2 fac — 0 (A)-2 (B)-Z (©) = dae (D) (E) none of these a ‘The equation 2” = 2 is satised when 2 is equal to: (A) infinity (B)2(C) YF (D) -VZ_(B) none of these 0 45. a1. 49. THE CONTEST PROBLEM BOOK : ces a ‘The sum to infinity of 4+ 2+ 5+ 5+ ++ ist 1 1 5 OM? Me; OF The graph of y = log z (A) cuts the y-axis (B) cuta all lines perpendicular to the 2-axie (©) cuts the z-axis (D) cuts neither axis (B) cuts all circles whose center is at the origin ‘The number of diagonals that can be drawn in a polygon of 100 sides (A) 4850, (B) 4950 (C) 9900 (D) 98 (B) 8800 In triangle ABC, AB = 12, AC =7, and BC = 10. Isides AB and ‘AC are doubled while BC remains the same, then: (A) the area is doubled (B) the altitude is doubled (C) the area is four times the original ares (D) the median is unchanged —(E) the area of the triangle is 0 A rectangle inscribed in a triangle has its base coinciding with the base ® of the triangle. If the altitude of the triangle is A, and the altitude = of the rectangle is half the base ofthe retangle, then: a ah th i Want = py Org, (2-4/8 (E)z = ‘A point is selected at random inside an equilateral triangle. From this nt perpendicular are dropped to each side, The sum ofthese perpen- ular is: (A) least when the point is the center of gravity of the triangle (B) greater than the altitude of the triangle (©) equal to the altitude of the triangle (D) one-half the sum ofthe sides of the triangle (H) greatest when the point isthe center of gravity ‘A triangle has a fixed base AB that is 2 inches long. ‘The median from A to side BC is 1} inches long and can have any postion emanating from A. The locus of the vertex C_ of the triangle is: (A) a straight line AB, 1 in. from A (B) ecircle with A’ as center and radius 2 in, (©) circle with A. as center and radius 3 in. (D) a citele with radius 3 in, and center 4 in. from B along BA (B) an ellipse with A as a focus (D) 7 (E) none ofthese ). A privateer discovers a merchantman 10 miles to leeward at 11:45 a.m, ‘and with a good breeze bears down upon her at 11 mph, while the mer- cchantman can only make 8 mph in her attempt to escape. After a two hhour chase, the top sail of the privateer is carried away; she can now make only 17 miles while the merehantman makes 15. The privateer will overtake the merchantman PROBLEMS: 1951 EXAMINATION n (A) 3:45 pm, (B) 3:30pm. (C) 5:00pm. (D) 2:45 pm. 1951 Examination Part 1 |. The per cent that Mf is greater than N, is: (4) OME My OOM = NY" (MEN (py MEN (4 OM +) A rectangular field is half as wide as itis long and is completely enclosed bby yards of fencing. The area in terms of = is we ox © wz me Ii the length of diagonal of a square is a + 6, then the area of the square is AG+)! Bit! Cate (D)KE+h) (B) none of these A barn with a flat roof is rectangular in shape, 10 yd. wide, 13 yd. long. and 5 yd. high. It is to be painted inside and outside, and on the eeiin Dut not on the roof or floor. The total number of sq. yd. to be painted is: (A) 360 (B) 400 (C) 490 (D) 590 (E) 720 . Mr. A owns a home worth $10,000. He sells it to Mr. B at a 10% profit ‘based on the worth of the house. Mr. B sells the house back to Mr. A ft a 10% loss, Then: (A) A comes out even (B) A makes $1100 on the deal (C) A makes $1000 on the deal (D) A loses $900 on the deal (B) A loses $1000 on the deal . The bottom, side, and front areas of a rectangular box are known, The product of these areas is equal to: {A) the volume of the box. (B) the square root of the volume (C) twice the volume (D) the square of the volume (B) the cube of the volume . An error of 02" is made in the measurement of a line 10° long, while an error of only .2" is made in # measurement of a line 100" long. In com- parizon with the relative error of the first measurement, the relative error of the second measurement is: (A) greater by 18 (B) the same (©) less (D) 10 times as great () correctly deseribed by both (A) and (D) 2 THE CONTEST PROBLEM BOOK 8. The price of an article is cut 10%. To restore it to its former value, the ‘new price must be inereased by: (A) 10% (B)9% (C)114% (D) 11% _(E) none of these answers 9. An equilateral triangle is drawn with a side of length a. A new equi- lateral triangle is formed by joining the mid-points of the sides of the first one. Then a third equilateral triangle is formed by joining the mid. points of the sides of the second; and so on forever. The limit of the sum of the perimeters ofall the triangles thus drawn is: (A) Infinite (B) 54a (C) 22 (D) 6a (B) dha 10, OF the following statements, the one that is incorrect is (A) Doubling the base of « given rectangle doubles the area, (B) Doubling the altitude ofa triangle doubles the are. (©) Doubling the radius of « given circle doubles the area. (D) Doubling the divisor ofs fraction and dividing its numerator by 2 changes the quotient. (©) Doubling a given quantity may make it less than it originally was. 11. The limit of the sum of an infinite number of terms in a geometric pro- gression is a/(1 — r) where a denotes the first term and —1 <1 <1 denotes the common ratio. The limit of the sum of their squares is: 7 Br Ea © 12, At 2:16 o'clock, the hour and minute hands of a clock form an angle of: (A) 30° (B) 5° (©) 224° (D) 7H (EH) 28" 13. A. can do a piece of work in 9 days. B is 50% more efficient than A. ‘The number of days it takes B to do the same piece of work is: (A) 13h (B) 44 (©) 6 (D) 3 CE) none of these answers (D) 42 CE) none of these a wy 14. In connection with proof in geometry, indicate which one of the following statements is incorrect (A) Some statements are accepted without being proved. (B) In some instances there is more than one correct order in proving certain propositions. (©) Every term used in a proof must have been defined previously. (D) It is not possible to arrive by correct reasoning at a true conclusion if, in the given, there is an untrue proposition. (2) Indireet proof can be used whenever there are two or more contrary propositions. 15, ‘The largest number by which the expression nt — n is divisible for all possible integral values of, is: M2 ®™3 Of M5 HE 16. 1. 18. 19. a. PROBLEMS: 1051 EXAMINATION 18 Part 2 If in applying the quadratic formula to # quadratic equation Se) = a2 + bo = 0, it happens that ¢ = 6¥/4a, then the graph of y = f(2) will certainly: (A) have a maximum —(B) have a minimum (©) be tangent to the z-axis (D) be tangent to the y-axis (B) lie in one quadrant only Indicate in which one of the following equations y is neither directly nor inversely proportional to =: Arty=0 (B)8y=10 (C)z=5y (D) a +y=10 F- v3 ‘The expression 2124 + az + 21 is to be factored into two linear prime binomial factors with integer coefficients. This can be done if a is: (A) any odd number —(B) some odd number (C) any even number (D) some even number (KE) sero A six place number is formed by repeating three place number; for example, 256,256, or 678,678, ete. Any number of this form is always exactly divisible by: (A) Tonly (B) only (©) 18 only (D) 101 (E) 1001 1. When simplified and expressed with negative exponents, the expression (e+ yet +) is equal to: Ort aeylt rt Bettrertyt Orv Ort Man Given: 2 > 0, y > 0, 2 > y and z #0. The inequality which is not always correct is: Azte>yt: Bz-2>y-2 Care Oia M5>5 Owre >. The values of a in the equation: logw(a# — 15a) = 2 are: (4) BE VBE (By 20,-5 (Cc) BA VBS (D) 200 (B) none of these ‘The radius of a eylindrical box is 8 inches and the height is 3 inches ‘The numberof inches that may be added to ether the radius or the height to give the same non-zero increase in volume (A) 1 (B) 54 (C) any number (D) non-existent (E) none of these “ 2 2. a. THE CONTEST PROBLEM BOOK at — 2077) oF (aye — 2 (By-2" (oy DE EDT 3 gine ‘The apothem of a square having its area numerically equa to its perimeter is compared with the apothem of an equilateral triangle having its area numerically equal to its perimeter. The frst apothem willbe: (A) equa tothe second (B) $ timesthe second (C) <7 imestheseond ‘when simplified () YF times the second) indeterminualy elated to the sxond fon 22 = 1) = (m+) = e Tn the equation #= AEC" +) 6 aye rots ar equal whe (Aym=1 (B)m = (C)m=0 (D) m= -1 (E)m= ~$ ‘Through a point inside a triangle, three lines are drawn from the vertices, to the opposite sides forming six triangular sections. Then: (A) the tangles are similar in opposite pairs (B) the triangles are congruent in opposite pairs (©) the triangles are equal in area in opposite pairs (D) three similar quadrilaterals are formed (B) none of the above relations is true The pressure (P) of wind on a sail varies jointly as the area (A) of the sail and the square of the velocity (V) of the wind. The pressure (on a square foot is 1 pound when the velocity is 16 miles per hour. The velocity of the wind when the pressure on a square yard is 38 pounds is: (A) 10% mph (B) 96 mph (C) 32 mph (D) 14 mph (E) 16 mph OF the following sets of data the only one that does not determine the shape ofa triangle is: (A) the ratio of two sides and the included angle (B) the ratios ofthe three altitudes (©) the ratios of the three medians (D) the ratio of the altitude to the corresponding base (I) two angles If two poles 20° and 80” high are 100* apart, then the height of the intersection of the lines joining the top of each pole to the foot of the opposite pole is: (A) 50" (B) 40” (©) 16" (D) 60" (1) none of these A total of 28 handshakes was exchanged at the conclusion of a party. Assuming that each participant was equally polite toward all the others, the number of people present was: (A) 14 (B) 2 (C) 55 (DS (HT 2, 36. 37, PROBLEMS: 1951 EXAMINATION 15 If AABC is inscribed in a semicircle whose diameter is AB, then AC + BC must be: (A) equal to AB (B) equal to ABY2 © (C) > ABVE (D) sABV2 (e) AB (@>b isread “a is equal to or greater than 8.”) 3. The roots of the equation 2t — 2x = 0 can be obtained graphically by finding the abscissas of the points of intersection of each of the following pairs of equations except the pai: @)y=2,y=% B)y=2-2%, y=0 Cy=azy (D)y=2— 241, y=1 (y= 2-1, y= 2-1 The value of 10!" is: (A) 7 (B) 1 (©) 10 (D) logus7 (EH) Jogs10 It or = c= b and or = a= d, then: Wane @i-f @sty-a+e We-y=e-e @e=9 Part 3 Which of the following methods of proving a geometric figure locus is not correct? (A) Every point on the locus satisfes the conditions and every point not on the locus does not satisfy the conditions. (B) Every point not satisfying the conditions is not on the locus and every point on the locus does satisfy the conditions. (©) Every point satisfying the conditions is on the locus and every point on the locus satisfies the conditions. (D) Every point not on the locus does not satisfy the conditions and every point not satisfying the conditions is not on the locus. (B) Every point satisfying the conditions is on the locus and every point not satisfying the conditions is not on the locus. ‘A number which when divided by 10 leaves a remainder of 9, when divided by 9 leaves a remainder of 8, by 8 leaves a remainder of 7, ete. down to where, when divided by 2, it leaves a remainder of 1, is: (A) 59 (B) 419 (C) 1359 (D) 2519 (E) none of these answers A rise of 600 fect is required to get a railroad line over « mountain. The ‘grade can be kept down by lengthening the track and curving it around the mountain peak. The additional length of track required to reduce the grade from 3% to 2% is approximately: (A) 10,000 ft. (B) 20,000 ft. (C) 30,000 ft. (D) 12,000 ft. (B) none of these 16 THE CONTEST PROBLEM BOOK 39, A stone is dropped into a well and the report of the stone striking the bottom is heard 7.7 seconds after itis dropped. Assume that the stone falls 16@ feat in ¢ seconds und thatthe velocity of sound fs 1,120 feet per second, The depth of the wells (A) 784 ft. (B) 342 ft. (C) 1568 ft. (D) 156.8 ft. (B) none ofthese ti -2+) (2-et+e+V¥ . (eee en F cnet (A) (+1) (B)G@ +1 (Ch (D) [le + IG — PF © (e- yp 41, The formula expressing the relationship between z and y in the table zp]? ]s]7)] 5] 6 yrol2 [el] (yy =2-4 (B)y=t-ar+2 Cy=2— 3420 Dyst—4 Bya2-4 eee ae (A)z=1 (BO<2r<1 (C)1t q (Opmant ©) p= -t (D) p= mH 1 Angle B of triangle ABC is trisected by BD and BE which meet AC at _D and_E respectively. Then: (a DE (gy D_B «) a, 2, 26, n. THE CONTEST PROBLEM BOOK . If /y = 3/4, then the incorrect expression in the following is: w@2t¥-7 pt -t @tteaF ae Me=3 © ‘The sides of a regular polygon of sides, n> 4, are extended to form ‘star. The number of degrees at each point of the star is: (a) 22 (gy BHAI oy = BIN ee a v (D) 180-2 (x) 180 On hypotenuse AB of a right triangle ABC a second right triangle ABD is constructed with hypotenuse AB. If BC = 1, AC = b, and AD = 2, then BD equals: (VEE (B)VE—3 (0) VFFI+2 (D)U+5 (BE) RES 1 Z=* = BF has rots which are numerically equal but of op- posite signs, the value of m must be: anb a+b ae ws wits @e wi wi . In the figure, itis given ‘that angle C = 90°, 4D = DB, DE 1. AB, AB = 20, and AC = 12. The € ‘rea of quadrilateral ADEC is: (a) 7 (B) 58} os @) 374 N : 8 (£) none of these ‘A powderman set a fuse for & blast to take place in 30 seconds. He ran ‘away at a rate of 8 yards per second. Sound travels at the rate of 1080 feet per second. When the powderman heard the blast, he had run ap- proximately: (A) 200 yd. (B) 352 yd. (C) 300 yd. (D) 245 ye.) B12 yd, ur (48) =, hen #44 cas (Al (2 ()O DM)3 6 ‘The ratio of the perimeter of an equilateral triangle having an altitude ‘equal to the radius of a cirele, to the perimeter of an equilateral triangle inseribed in the cirele i: ML2 BLS Onve (D) V2 (B23 al. 82. 34. PROBLEMS: 1052 EXAMINATION a In the table shown, the formula relating 2 and y is: ose fceas) arises] cee aee ieee i [errs p's) atest: Myade-1 Byyse-etr4+2 Oy=Ferti (D) y= G+2+ De — 1) (EB) none of these In a circle of radius § units, CD and AB are perpendicular diameters. Acchord CH cutting AB at K is 8 units long. The divided into two segments whose dimensions are: (A) 125,875 (B) 275,725 (C)2,8 (D) 4,6 (B) none of these ‘When the sum of the first ten terms of an arithmetic progression is four times the sum of the first five terms, the ratio of the first term to the ‘common difference is: @L2 (B21 (Ci Ws (Bt Given 12 points in & plane no three of which are collinear, the number of lines they determine is: (A) 24" (B) 5 (©) 120 (D) 66 (EH) none of these K. takes 30 minutes less time than M to travel a distance of 30 miles. K travele } mile per hour faster than Mf. If x is K's rate of speed Snail pet hur then K's ume forthe dane cy 2A) (m2) oy D2 wy 3. A circle and a square have the same perimeter. Then: (A) their areas are equal (B) the area of the circle is the greater (©) the area of the square is the greater (D) the area of the circle is x times the area of the square (B) none of these ‘The price of an article was increased p%. Later the new price was de- creased p%. Ifthe last price was one dollar, the original price was: 1 Pr yee) AGF (0) onedotiar (D)1- ae — 10,000 © wit a i v2 ith a rational denominator, the expression —-—V! wit 7 tor, the exresion eV is equivalent to: (a StVE4VIE Gm VI-2tViO (c 2+VO4 VI (B) 2¢VER VIO (1) none of these 38. 31. 39, 4 42. THE CONTEST PROBLEM BOOK Part 3 Tote continue at = = =, the val of 2? i tke obs (4)-2 (BO (C3 = (E)- ‘Two equal parallel chords are drawn & inches apart in circle of radi 8 inches. The ara ofthat pat ofthe circle that lies between the chords (A) 21} — 32. (B) 8249/3 + 21x (C) 32/3 + 424r (D) 16/3 + 429n (EB) 429 “The area of a trapezoidal fld in 1400 square yards. Its altitude is 50 ‘yards. Find the two bass, ifthe number of yards in each base isan in- teger divisible by 8. The number of solutions to this problem is: (A) none (B) one (C) two (D) three (E) more than three Tf the perimeter of a rectangle is p and its diagonal is d, the difference ‘between the length and width of the rectangle is: (a) WERE py VET oi VEEP y VRE ). In order to draw a graph of f(z) = ax* + br + ¢, a table of values was constructed. These values of the function for a set of equally spaced inereasing values of 2 were 3644, 3069, 4006, 4227, 4356, 4489, 4624, sand 4761. The one which is incorrect is (A) 4096 (B) 4356 (C) 4489 (D) 4761 (E) none of these Increasing the radius of a cylinder by 6 units increases the volume by ¥y cubic Units, Increasing the altitude of the eylinder by 6 units also increases the volume by y cubic units. If the original altitude is 2, then the original radius is: (2 B)4 C6 Wor 8 Let D represent a repeating decimal. If P denotes the r figuros of D which do not repeat themselves, and Q denotes the « figures which do repeat themselves, then the incorrect expression is: (A) D = PQQQ---(B) 10'D = P.QQ0- (© 10D = PQ.QQ0 ++. (D) 1000" = 1) = QP = 1) (&) 10-10"D = POQ.000 ‘The diameter of a circle is divided into n equal parts. On each part a semicircle is constructed. As becomes very large, the sum of the lengths of the arcs of the semi-circles approaches a length: (A) equal to the semi-circumference of the original circle (B) equal to the diameter of the original circle (©) greater than the diameter but less than the semi-circumference of the original ci PROBLEMS: 1952 EXAMINATION 2 (D) that is infinite () greater than the semi-cireumference but finite 44. If an integer of two digits is k times the sum of its digits, the number formed by interchanging the digits is the sum of the digits multiplied by. A@-hH (B)0O-k O(-k Mk-) (H)k+) 48. If @ and b are two unequal positive numbers, then: 2b ath Zab a+b Feb? VPS B) Vab > > 2ab ath a+b © Bt va my ated 2ab ® tf" > va > 28, 46, The base of anew rectangle equals the sum of the diagonal and the greater side of a given rectangle, while the altitude of the new rectangle equals ‘the difference of the diagonal and the greater side of the given rectangle. ‘The area of the new rectangle is: (A) greater than the srea of the given rectangle (B) equal to the area of the given rectangle (©) equal to the area ofa square with its side equal to the smaller side of the given rectangle (D) equal tothe area of a aquare with its side equal to the greater side of the given rectangle (B) equal to the area ofa rectangle whose dimensions are the diagonal and shorter side ofthe given rectangle 47. In the set of equations 2" = y", 2' = 24%, 2+ y +2 = 16, the integral roots inthe order 2, 9 2 are: (A)3, 4,9 (B)9,-5,12 (C) 12, -5,9 (D) 4,3,9 (E) 4,9,3 48, Two cyclists, k miles apart, and starting at the same time, would be together in r hours if they traveled in the same direction, but would pass each other int hours if they traveled in opposite directions. The ratio ofthe speed of the faster cyclist to that of the slower is: wrt! @t, wtt! wt ttt 49, In the figure, CD, XB and BP are one-third of their respective sides. Tt follows that AN: NN ND , A fand similarly for lines BE and CP. ‘Then the aren of triangle NNN is: (A) te & ABC *< @ #a 4a : QO (© $4.aBc (D) $4 aac | 5 (B) none of these m THE CONTEST PROBLEM BOOK 50. A line initially 1 inch long grows according to the following law, where the first term is the initial length, 1 faa rena reivittetvesded vase... If the growth process continues forever, the limit of the length of the Tine is Ae Bt OF Ow+v} Hia+vH 1953 Examination Part 1 1. A boy buys oranges at 3 for 10 cents. He will sell them at 5 for 20 cents. In order to make a profit of $1.00, he must sel: (A) 67 oranges (B) 150 oranges’ (C) 200 oranges, (D) an infinite number of oranges (E) none of these 2. A refrigerator is offered for sale at $250.00 less successive discounts of 20% and 15%. The aale price of the refrigerator is: (A) 85 % less than $250.00 _ (B) 65°% of $250.00 (©) 77% of $250.00 (D) 68% of $250.00 (E) none of these 3, The factors of the expression 2* + y# are: Aetoe-D Beto Ces ee" +y%) (D) (+ Wee — ty) CB) none of these 4. The roots of (2! + 8 + 16)(4 — z) = 0 are: (0 (B)0,4 (C)0,4,-4 (D) 0,4, -4, -4 (B) none of these 5. If log = 2.5, the value of 2 is: (A) 90 (B) 36 (C) 366 (D) 0.5 (BE) none of these 6. Charles has 5¢ + 1 quarters and Richard has q + 5 quarters. The difference in their money in dimes is: (A) 10@ - 1) (B) 4g — 4) ()KQ— 1) (D)AG-1) (B) none of these 1 Meteo Vet 7 7. The fraction VE EH = Gt = o/s reduces to: 2a" 2a od OO Bats Oem Mera Oars 10. UL 12. 13. M4. 15, PROBLEMS: 1053 EXAMINATION % . The value of z at theintersection of y = 8/(2" + 4) and x+y = 2 is: (A) 2+ V8 (B)-2— V5 (CO (D) 2 (B) none of these ‘The number of ounces of water needed to reduce 9 ounces of shavin lotion containing 50% alcohol to a lotion containing 30% aleohol is: 3 (4 (C5 (D6 (7 ‘The number of revolutions of wheel, with fixed center and with an ‘outside diameter of 6 feet, required to cause a point on the rim to go one rile is: (4380 M2 (OS (>) oy noect hoe ‘A running track is the ring formed by two concentric circles. It is 10 {feet wide. The circumferences of the two circles differ by about: (A) 10 feet (B) 30 feet (C) OO feet (D) 100 feet (B) none of these ‘The diameters of two circles are 8 inches and 12 inches respectively. The ratio of the area of the smaller to the area of the larger circle ea Tees (A) 3 (BFC) (D)Z (EB) none of these ‘A triangle and a trapezoid are equal in ares. They slso have the same aktitude. If the base ofthe triangle is 18 inches, the median ofthe trapezoid (A) 36 inches _(B) 9 inches (C) 18 inches (D) not obtainable from these data (E) none of these ven the larger of two circles with center P. and radius p and the smaller with center Q and radius q. Draw PQ. Which ofthe follow- ing statements is false? (A) p= 9 can be equal to FO (B) p+ q can be equal to PQ (©) p+ canbe less than PO (D) p—2 eanboless than PQ () none of these A circular piece of metal of maximurp site is cut out of « square pioce and ‘then a square piece of maximum size is cut out of the circular piece. The ‘otal amount of metal wasted is: (A) 2} the area of the original square (B) § the area of the original square (C) 4 the area ofthe cireular pioce (D) 3 the area of the circular pioce ) none of those 16, 17. 18 10. 21 25, THE CONTEST PROBLEM BOOK Part 2 Adams plans a profit of 10% on the selling price of an article and his ‘expenses are 15% of sales. The rate of mark-up on an article that sells for $5.00 is: (A) 20% (B) 25% (©) 30% (D) 333% (E) 35% ‘Aman has part of $4500 invested at 4% and the rest at 6%. If his annual return on each investment is the same, the average rate of interest which he realizes on the $4500 ia: (A) 5% (B)4.8% (C)52% (D) 4.6% (EB) none of these One of the factors of 34+ 4 Ae+2 Br+l O#-2+2 Dz (B) none of these In the expression 94, the values of x and y are each decreased 25 %; the value of the expression is: (A) decreased 50% —(B) decreased 75% (© decreased 37/64 ofits value (D) decreased 27/64 of its value (B) none of these it yet den Ate ae bet 1-0 beams Wey ty-]=0 BaAy+y-3=0 Ox¥+y-4)=0 (D)e+y-6) =0 (B) none of these If logy (2? — 3 + 6) = 1, the value of 2 is: (A) 1002 (B)4or—2" (C)Sor—1 (D) dor =1 (®) none of these ‘The logarithm of 27 79 +/9 to the base 3 is: (A) 8} (B)4E (C)5 (D3 (B) none of these ‘The equation VF F16 — 5 has: vz +10 (A) an extraneous root between —5 and’~1 (B) an extraneous root between —10 and —6 (©) a true root between 20 and 25 (D) two true roots (E) two extraneous roots If a, b and are positive integers less than 10, then (10a + 6)(10a + ¢) equals 1000(a + 1) + be if @b+e=10 (B)b=c (atb=10 (Dab @atb+e=10 In a geometric progression whose terms are postive, any term is equal to the cum of the next two following terms. Then the common ratio is: 31 PROBLEMS: 1953 EXAMINATION 2” WB v3 2 vi 1 -v3 (A)1(B) about (5) 3. The base ofa triangle is 16 inches. Two lines are drawn parallel to the base, terminating in the other two sides, and dividing the triangle into three equal areas. The length of the parallel clase to the base is: (A) bY inches (B) 10 inches (C) 4/3 inches (D) 7.5 inches, (E) none of these ‘The radius of the first circle is 1 inch, that of the second 4 inch, that of the third } inch and so on indefinitely. ‘The sum of the areas of the circles (a) SE (B)13r (C)2x (D) AT CB) none of these In triangle ABC, sides a, b and ¢ are opposite angles A, B and C respectively. AD bisects angle A and meets BC at D. Then if 2 = CD and y = BD the correct proportion is: (A)z/a=a/b+o (B)z/b=a/ato (C)yle=efb+0 Dye afb+0 (E)z/y =e |. The number of significant digite in the measurement of ‘he side of square whose computed area is 1.1025 square inches to the nearest ten- thousandth of a square inch is: 2 Bs ©4 OS HI ‘A house worth $9000 is sold by Mr. A to Mr. B at a 10% loss. Mr. B sells the house back to Mr. A at a10% gain. The result of the two tran actions is (A) Mr. A breaks even _(B) Mr. B gains $900 (C) Mr. A loses $900 (D) Mr. A loses $810 (EE) Mr. B gains $1710 ‘The rails on a railroad are 30 feet long, As the train passes over the point ‘where the rails are joined, there is an audible click. The speed of the train in miles per hour is approximately the number of clicks heard in: (A) 20 seconds (B) 2 minutes (C) 15 minutes (D) § minutes (B) none of these Each angle of a rectangle is triected. The intersections of the pairs of trisoctors adjacent to the same side always form: (A) asquare (B) arectangle (C) a parallelogram with unequal sides (D) athombus —(E) a quadrilateral with no special properties ‘The perimeter of an isosceles right triangle is 2p. Its area is: A+ V2 B@-VIp_ (CB - 22 MD) =2V2e (G+ 2VIP THE CONTEST PROBLEM BOOK 3A. If one side of a triangle is 12 inches and the opposite angle is 30 degrees, 35. 36. 38. 39, 41 2, then the diameter of the circumscribed circle is: (A) 18 inches (B) 30 inches (C) 24 inches (D) 20 inches (B) none of these ) i f@) = 4 , then fz + 2) equals (A) 2) +2) (B) (e+ 2) (C) xe + 2flz)_ (D) EF DME+D a(x) z+? Part 8 Determine m so that 4: — Gr + m is divisible by x — 3. The ob- tained value, m, is an exact divisor of: (12 B® (3 (Das Ho ‘The base of an isosceles triangle is 6 inches and one of the equal sides 12 inches. The radius of the circle through the verties of the triangle 5 w4va_ ©3VE_(D)6V3_ CB) none thew Uf f(a) = a~2 and Fla,b) =H +a, then FI, f(4)] i We-i+7 BB O7 Os (iu The product, log, b-logy a is equal to: (A)1 (Ba (©) (Dj ab (E) none of these ). The negation of the statement “all men are honest,” is: (A) no men are honest (B) all men are dishonest (©) some men are dishonest (D) no men are dishonest (2) some men are honest AA irl’ camp is located 800 rods from a straight road. On this road, a boys’ camp is loeated 500 rods from the ies’ camp. It is desired to 4 canteen on the road which shall be exactly the same distance from each camp. The distance ofthe cantesn from each of the camps is (A) 400 rods (B) 250 rods (C) 87.5 rods (D) 200 rods (B) none of these ‘The centers of two circles are 41 inches apart. ‘The smaller circle has © radiua of 4 inches and the larger one has a radius of 5 inches. The length ‘of the common internal tangent is: (A) 41 inches (B) 39 inches (C) 39.8 inches (D) 40.1 inches (B) 40 inches 43. 4. 45. 46. a 49. PROBLEMS: 1953 BXAMINATION ~ If the price of an article is increased by per cent. p, then the decrease in per cent of sales must not exceed d in order to yield the same income, ‘The value of is: a eo ee ery yy coed Om ®t; OF OFA @5? In solving a problem that reduces to a quadratic equation one student ‘makes a mistake only in the constant term of the equation and obtains 8 and 2 for the roots. Another student makes a mistake only in the coefi- cient of the first degree term and finds —9 and —1 for the roots. The 0 (B+ 49-0 (#12 +16=0 © (EB) none of these ‘The lengths of two line segments are a_units and 6 units respectively. Then the correct relation between them Sts ya cm Sth cya Et). va SF) 5 yas St) = vas Instead of walking along two adjacent sides of a rectangular field, a boy took a short-cut along the diagonal of the field and saved a distance equal to 1/2 the longer side. Te ratio of the shorter side of the rectangle to the longer side was: 1 2 1 3 wy? ws me (E(w op} If = is greater than zero, then the correct relationship is: (A) log (1 + 2) = 2/(U + 2) (B) log(l + 2) < 2/0 + 2) (©) log (1 +2) > 2 (D) log (1+ 2) <2 (E) none of these 3. If the larger base of an isosceles trapezoid equals a diagonal and the smaller base equals the altitude, then the ratio of the smaller base to the larger base is: (1/2 (B)2/B Cad (D) 3/5 (H) 2/5 ‘The coordinates of A, B and_C are (5,5), (2, 1) and (0, k) respectively. ‘The value of & that makes AC + BC as small as possible is: ws B44 O% OH OA One of the sides of a triangle is divided into segments of 6 and 8 units by the point of tangeney of the inscribed circle. Ifthe radius of the cirele is 4, then the length of the shortest side of the triangle is (A) unite (B) 19 units (C) 14unite (D) 15 units (E) 16 unite |. The square of 5 — iF = 35 THE CONTEST PROBLEM BOOK 1954 Examination Part 1 (Wt = SVP = (B) Ov D6-y ©) ¥ - VFB The equation 22, = ET 4 A=" 4 1 = 0 canbe tranfomed . A regular hexagon is inseribed in a circle of radius 10 inches. Tts area is by eliminating fractions to the equation 2* — Sr +4 = 0. The roots of the latter equation are 4 and 1. Then the roots of the first equation are: (A)4and 1 (B) only 1 (C) only 4 (D) neither 4 nor 1 (B) 4 and some other root If 2 varies as the cube of y, and y varios as the fifth root of 2, then varies as the nth power of z, where 1 is: 1 3 3 (ig (DF (OZ Ws (Es If the Highest Common Divisor of 6432 and 192 is diminished by 8, it will equal: (A) 6 (B)G (C)-2 (D)3. (HA (A) 1500/3 sq. it (B) 150 sq. in. (C) 254/3 sq. in. (D) 600 sq. in. (B) 3004/3 sq. in. he vaeat (B)o+ (gh)! = oe = (aay i WH Bie Ol OF Bx A housewife saved $2.50 in buying a dress on sale. If she spent $25 for the dress, she saved about: (A)8% | (B)9% (C)10% (D) 11% —(B) 12% 3. The base of a triangle is twice as long as aside of a square and their areas are the same. Then the ratio of the altitude of the triangle to the side of the square is 1 1 (QP 5 (]1 (2 Hs A point P is outside a circle and is 13 inches from the center. A secant from P cuts the circle at Q and R so that the external segment of the secant PQ is 9 inches and QR is 7 inches. The radius of the circle as a (Cer (D6 ET PROBLEMS: 1954 EXAMINATION a 10. Tho sum of the numerical coefficients in the expansion of the binomial (a+ HF is: (A)s2 B16 (C64 (D) 48 (E)T 11. A merchant placed on display some dreses, ench with marked price. He then posted a sign “t off on these dresses.” The cost of the dresses vras J of the price ab which he actually sold them. Then the ratio of the cost to the marked price was: a eer er wi ME ©} Mz we 12, The aiton ofthe equtons 2 = 297 7 are TIS EZ} © theres no solution y=12 ®yn0 (D) there i an olmited numberof solutions cE) = § 13. A quadrilateral is inscribed in a circle. If angles are inscribed in the four ‘aes cut off by the sides of the quadrilateral, without intersecting the sides betwoen vertices, their sum will be: (A) 180° (B) 540° (C) 360° (D) 450° (B) 1080" Fod @p+a 15. Log 125 equals: (A) 100 10g 1.25 (B) 5 log 3 (C) 3 og 25 (D) 3 — 3log2 (B) (log, 25)(log 5) Part 2 16. If flz) = 52 — 22 = 1, then fe +h) — flz) equals: (A) 5 — 2k (B) Ozh — dr +2 (C) 102k — Be — (D) A(loz + 5h — 2) (E) Bh 17. The graph of the function f(z) = 22 — 7 goes: (A) up to the right and down to the left (B) down to the right and up to the left (©) upto the right and up to the left (D) down to the right and down to the left (B) none of these ways. 18. 19, a 23, THE CONTEST PROBLEM BOOK OF the following sets, the one that includes all values of = which will satisfy 2r— 3 > 7 zis: 10 10 (z>4 B2<2 Cr-P Wr>B 3 If the three points of contact of a circle inseribed in a triangle are joined, the angles of the resulting triangle: (A) are always equal to 60° (B) are always one obtuse angle and two unequal acute angles (©) are always one obtuse angle and two equal acute angles (D) are always acute angles (B) are always unequal to each other ‘The equation 2+ 6:° + Iz + 6 = 0 has: (A) no negative real roots (B) no positive real roots (C) no real roots (D) 1 positive and 2 negative roots (F) 1 negative and 2 positive roots ‘The roots of the equation 2/ + 22" ‘can be found by solving: (A) 16 — 92 +10 (B) 4x ~ 25+ 4= 0 (ae — 244-0 (D221 +2 (B) det — %r—4 = 0 (B)z<0 at 4s Theexprenion Gis — 2) ~ + Ne — 2) cannne be evaluated for z= -1 of x= 2, since division by zoro is not allowed. For other values of 2: (A) the expression takes on many different values (B) the expression has only the value 2 (©) the expression has only the value 1 (D) the expression always has a value between —1 and +2 (E) the expression has a value greater than 2 or less than —{ i the marin mate on an arti coting © lan and eng for $ datas in M = 16, then the margin igen by 1 (aM {8 Bais (om 1 ae om os @M- its ‘The values of & for which the equation 20° — kr + z+ 8 = 0 will Ihave real and equal roots are: (A) 9 and -7 (B) only —7 (C)9and7 (D) Sand 7 (E) only 9 ‘The two roots of the equation a(b — c)zt + W(e — a) + ofa = b) =O are 1 and: a) ab (yab-9 (py a—o we ~ ao—=9 @ea=h Obey aoe “ 26, 28, 29. 31 2, PROBLEMS: 1054 EXAMINATION 3 ‘The stright line AB is divided at C 20 that AC = 3CB. Circles are deseribed on AC tnd CB as diameters and a common tangent meets AB produced at D. Then BD equals: (A) the diameter ofthe smaller circle (B) the radius ofthe smaller cirele (©) the radius of the larger circle (D) CBV3 (EB) the diference of the two radil A right circular cone has for its base a circle having the same radius as a sven sphere. The volume of the cone is one-half that of the sphere. The ratio of the at negara ana OM; BD, ©} DWT H4/z m4 9 mr = 5 ond P=, the value of (@)-5) @)-H (- (OH ©) -4 If the ratio of the legs of a right triangle is 1:2, then the ratio of the corresponding segments of the hypotenuse made by a perpendicular ‘upon it from the vertex i: Ala Biv (C12 (uve (Ls A and B together can do a job in 2 days; B and C can do it in four days;and A and C in 24 days. The number of days required for A to do the job alone is: ®1 B3 ©6 Me wz In triangle ABC, AB = XC, angle A = 40°. Point O is within the triangle with angle OBC = angle OCA. The number of degrees in angle BOC is (A110 B)35 (40) 55 (H) 70 ‘The factors of zt + 64 are: E+ BE +H 8) (Ot + 22+ Ht - Br + 16) (D) (a — Ae + Bat — Ae — 8) (B) de $F + Ae + 8) ‘A bank charges $6 for & loan of $120. The borrower receives $114 and repays the loan in 12 installments of $10 & month. The interest rate is ‘approximately: (5% —B)O% (C)7% (D)9% (B) 15% The fraction, 1/3 ee ee ea (© ste thao by hy (D) in greater than 0.38883383 by 5-755 ee tee au THE CONTEST PROBLEM BOOK 235. In the right triangle shown the sum of the distances BM and MA is equal to the sum of the distances ‘BC and CA. it MB = then z equals: ™ Ad A “ aha 8 (B)d-h © i 4 D)hta- Va ©) VBFEH A 7 ‘ Z Part 3 38. A boat has & speed of 15 mph in stil water. In a stream that has a current of 5 mph it travels a certain distance downstream and returos ‘The ratio of the average speed for the round tip to the speed in sill water is: at we 8 Tete, 7 ©F wm? & 87. Given triangle PQR with RS bisecting angle R, PQ extended to D and angle n fright angle, then: (A) Am = Hap — 40) (B) 4m = Map + 49) © 44 = iat ap) (©) 4d = dam (B) none of these is correct 38. If log2 = 3010 and log3 = 4771, the value of 2 when 3*¥ = 135 is approximately: (A)5 (B)LAT (C167 (D178 () 1.68 39. The locus of the mid-point of a line segment that is drawn from a given external point P to a given circle with center O and radius r, is: (A) a straight line perpendicular to. PO (B) a straight line parallel to PO (©) a circle with center P and radius r (D) a circle with center at the midpoint of PO and radius 2r () a circle with center at the mid-point of PO and radius dr a. 43. a (04) PROBLEMS: 1954 EXAMINATION 35 ho gu w2¥8 @svi 0 WIVT Bov3 ‘The sum of all the roots of 42% — St — 68x — 9 = 0 ist 8 2 (C)-8 (D)-2 (HO ‘Consider the graphs of (1) y = 2* — $2 + 2and (2) y= 2° + dz + 20n the same st of ate. These parabolas have exacly th same shape Then (A) the graphs coincide (B) the graph of (1) is lower than the graph of (2) (C) the graph of (1) is to the left of the graph of (2) (D) the graph of (1) is to the right of the graph of (2) (E) the graph of (1) is higher than the graph of (2) ‘The hypotenuse of a right triangle is 10 inches and the radius of the in- scribed circle is 1 inch. The perimeter of the triangle in inches is: (A) 15 (B)22, (C24 (D) 26 (E) 30 . A man born in the frst half of the nineteenth century was x yoars old in the year 2°. He was born in: (A) 1819 (B) 1825 (©) 1812 (D) 1836 (E) 1806, In & rhombus ABCD line segments are drawn within the rhombus, parallel to diagonal BD, and terminated in the sides of the rhombus. A raph is drawn showing the length of s segment as function ofits die tance from vertex A. The graph is: (A) a straight line passing through the origin (B) a straight line cutting acros the upper right quadrant (©) ‘wo line segments forming an upright. (D) two line segments forming an inverted V, (4) (B) none of these Tne diagram if points A, B, C are pointsoftangeney,then 2 equal: we «st ox m2 ” Big 36 THE CONTEST PROBLEM BOOK 47. Avthe midpoint of ine segment AB which is p unite long, a perpendicu- lar MR is erected with length units. An arcis described from with ‘radius equal to 448, meeting 4B at T. Then AT and TB are the roots of * WO tterte @) #-pte Ortme-e | Ne (D) # pee (=) #-prtg =0 w ek 48. A train, an hour after starting, meets with an accident which det half hour, after which it proceeds at 4 of its former rate and arrives 34 hhours late. Had the accident happened 90 miles farther along the line, it ‘would have arrived only 3 hours Inte. The length of the trip in miles was: (A) 400 (B) 465. (C) 600 (D) 640 (B) 580 49, The difference of the squares of two odd numbers is always divisible by B.If a> 6, and 20+ 1 and 2 +1 are the odd numbers, to prove the given statement we put the difference of the squares in the form: (A) Ga + 1 — (2+ 1% (B) da — 488 + 4a — 4b (©) lata + 1) ~ b+ 1) (D) 4a — a+b +1) ®4@ +a-H =) 50, The times between 7 and 8 o'clock, correct to the nearest minute, when the hands of a clock will form an angle of 84 degrees are: (A) 7:28 and (B) 7:20 and 7:50 (C) 7:22 and 7:58, (D) 7:28 and (8) 7:21 and 7:49 1955 Examination Part 1 1. Which one of the following is not equivalent to 0,000000375? (A) 375 X 10-7 (B) BE x 107 (©) 375 X 10" (D) # x 107 () bx 10 2. The smaller angle between the hands of a clock at 12:25 pam. is (A) 13230" (B) 137°30" (©) 150° (D) 13792" (BE) 137° 3 If each number in a set of ten numbers is increased by 20, the arithmetic mean (average) of the original ten numbers: (A) remains the same (B) is increased by 20 (C) is imereased by 200 (D) is increased by 10 (E) is increased by 2 10, ML. 12, 13. PROBLEMS: 1955 BXAMINATION a 2 ‘The equality 25 is satisfied by: (A) no real values of 2 (B) either z= 1 or 2=2 (C)only 2=1 (D) only z=2 (only z=0 1 varies inversely as the square of x. When y = 16, 2 = 1. When == 8, y equals: (A)2) (B)12B (C64 (D)A_ (B 1024 ‘A merchant buys a numberof oranges at 3 for 10f and an equal number ‘5 for 204. To “break even” he must sell all al: (A) 8for 304 (B)Sfor 1l¢ (C) Bfor 18¢ (D) 11 for 40¢ (B) 18 for 504 If a worker receives a 20 per cent cut in wages, he may regain his original pay exactly by obtaining raise of: (A) 20 per cent (B) 25 per cent (©) 224 per cent (D) $20 (E) $25 . ‘The graph of 2° — 4y? = 0: (A) is @ hyperbola intersecting only the z-axis (B) is @ hyperbola intersecting only the y-axis (C) isa hyperbola intersecting neither axis (D) is a pair of straight lines (E) does not exist A circle is inseribed in a triangle with sides 8, 15, and 17, ‘The radius of the cirele is: (6 (B)2 (5 (D3 (HT How many hours does it take a train traveling at an average rate of 40 ‘mph between stops to travel a miles if it makes » stops of m minutes each? (a) S24 2m"(B) 3a + 2mm (c) HIM (py atm 190 12 2) + 40mn (ey £4 fone ‘The negation of the statement “No slow learners attend this school, (A) All slow learners attend this school (B) All slow learners do not attend this achool. (©) Some slow learners attend this school (D) Some slow learners do not attend this school (B) No slow learners attend this school. ‘The solution of V/5z—1 + VF—T =2 is: (A)ra%r=1 Br=F (C)z=2 (D)r=1 Hr=0 ote ‘The fraction SF is equal to: (A)ot—b! Byatt (Chatzet (DHE (a — 6 38 THE CONTEST PROBLEM BOOK 14. The length of rectangle R is 10 per cent more than the side of square 'S. The width of the rectangle is 10 per cent less than the side of the square. The ratio of the areas, RS, is: (A) 99: 100. (B) 101: 100 (©) (©) 201 : 200 15. The ratio of the areas of two concentric circles is 1 : 3. If the radius of the smaller is +, then the difference between the radii is best approxi- mated by: (A) 0dr (B) 073° (C) 0.75 (D) 0.73" (B) 075° (D) 199 : 200 Part 2 3 are (3 2 (0 © aysniemanber (@) mesingae 16, The value of when a= 4 and b= —4 is: 17. Af log — 5 log 3 = 2, then x equals: (A) 125 (B) O81 (C) 243 (D) 08 CE) either 03 or 1.25 18, The diseriminant of the equation 2* + 22/5 +3 = 0 is zero. Henes, ite root are: (A) real and equal (B) rational and equal (C) rational and unequal (D) irrational and unequal (F) imaginary 19. Two numbers whose sum is 6 and the absolute value of whose difference in 8 are root ofthe equation: (@)2-6+7=0 @B#—6r-7=0 Ort or- D)#-Ge+8=0 C)#46r-7=0 20, The expresion V25=7 + 5. equals zero for: (A) no real or imaginary values of ¢ (B) no real values of ¢, but for some imaginary values (C) no imaginary values of t, but for some real values D)t=0 Ht 5 21, Represent the hypotenuse of a right triangle by ¢ and the area by A. ‘The altitude on the hypotenuse is: 4 2A 4 4 4 w4 wm wz Ms (Hs 22, On a $10,000 order a merchant has a choice between three successive discounts of 20%, 20%, and 10% and three successive discounts of 40%, 5%, and 5%. By choosing the better offer, he ean save: (A) nothing at all (B) $400 -(C) $330 (D) $345 (1) $360 23, In checking the petty cash clerk counts q quarters, d dimes, n nickels, and ¢ cents. Later he discovers that x of the nickels were counted as quarters and 2 of the dimes were counted as cents. To cor- rect the total obtained the clerk must: PROBLEMS: 1955 AMINATION 39 (A) make no correction (B) subtract 11¢ (C) subtract L1z¢ (D) add tix¢ (Badd 2¢ (B) always decreases as z decreases to 1 (C) cannot equal 0 (D) has a maximum value when 2 is negatiy (B) has « minimum value of —10 25, One of the factors of rt + 20% + 9 is: @ze+3 Bz+1 Ox2-3 M)e-a2—- (B) none of these 26. Mr. A owns a house worth $10,000. He sels it to Mr. B at 10% profit. ‘Mr. B sells the house back to Mr. A at a 10% loss. Then: (A) A comes out even (B) A makes $100 (C) A makes $1,000 (D) B loses $100 (EB) none of these is correet 27. It r and s are the roots of 2! ~ pr +g = 0, then #* + equals: Arty Br-% Or+e DP-F OP 28. On the same set of axes are drawn the graph of y = ar + br te and the graph of the equation obtained by replacing by —2 in the given equation. If 8 740 and c #0 these two graphs interest: (A) in two points, one on the z-axis and one on the y-axis {B) in one point located on neither axis (C) only atthe origin (D) in one point on the z-axis (8) in one point on the yan 29, In the figure PA is tangent to semicircle SAR; PB is P ‘tangent to semicircle RBT; SRT is straight line; the ares are indicated in the figure. Angle APB is meatured by: s, (a) Ha ~ 0) BHet) ©-a) -@- @)a-b a+b 50. Each_of the equations 32-2 = 25, (2x — 1) = (— 1), Ve=7 = VE=1 has: (A) two integral roots (B) no root greater than 3 (C) no root zero (D) only one root (B) one negative root and one positive root 3 33, 36. 37, THE CONTEST PROBLEM BOOK ‘An equilateral triangle whose side is 2 is divided into a triangle and a trapezoid by a line drawn parallel to one of its sides. If the area of the trapezoid equals one-half of the area of the original triangle, the length of the median of the trapezoid OM vi 2+ vi (2+ (mH? If the discriminant of az* + 26 +.¢ = 0 is zero, then another true statement about @, }, and ¢ isthe (A) they form an atithmetie progression (B) they form a geometric progression (© they are unequal (D) they are all negative numbers (B) only } is negative and a and ¢ are positive Henry starts a trip when the hands of the clock are together between 8 ‘am. and 9am. He arrives at his destination between 2 p.m. and 3 p.m. ‘when the hands of the clock are exactly 180° apart. ‘The trip takes: (A) hr. (B) 6 hr. 439 min. (C) 5 hr. 167 min. (D) 6 hr. 30 min. (B) none of these .A.G-inch and 18-inch diameter pole are placed as in the figure and bound together wth wire The length of ogee ts ae (A) 12V3 + 16 (123 + Te é) (© 12/3 + Me (B12 toe ae 5. Three boys agree to divide a bag of marbles in the following manner. ‘The first boy takes one more than half the marbles. The second takes a third of the number remaining. The third boy finds that he is left with ‘twice as many marbles as the second boy. The original number of marbles: (A) is 8 or 38 (B) cannot be determined from the given data (©) is 20 or 26 (D) is 14 or 32 (E) is none of these Part 3 ‘A cylindrical oil tank, Iying horizontally, has an interior length of 10 feet and an interior diameter of 6 feet. If the rectangular surface of the oll has an area of 40 square feet, the depth of the oil is: A V5 (B)2V5 (C)3- V5 (D)3+ VE (© either 3 — V5 or 3 + vi A three-digit number has, from left to right, the digits h, & and w with A> u. When the number with the digits reversed is subtracted from the original number, the units’ digit in the 39, al. 2, PROBLEMS: 1955 EXAMINATION a digit, from right to left, are: (A) Sand 9 (B)9 and 5 (C) impossible to tell (D) 5 and 4 (B) 4 and 5 Four positive integers are given. Select any three of these integers, find their arithmetic average, and add this result to the fourth integer. Thi the numbers 29, 23, 21 and 17 are obtained. One of the original integers @is BA (Oz 2% (iT If y= 2+ prt q, then if the least possible value of y is zero, ¢ is equal to 2 ow? wm-2 me- cao (HF (2 ~w)-2 HE —¢ 2 jone +5 gag & it Hf bd, the fractions SE F4 and 3 are unequal if (a=c=1 and 240 (B)a=b=0 (Ca o D)z=0 Cad = be ‘A train traveling from Aytown to Beotown meets with an accident after 1 hi, It is stopped for } hr., after which it proceeds at four-fifths of its usual rate, arriving at Beetown 2 hr. late. If the train had covered 80 miles more before the accident, it would have been just 1 hr. late. The usual rate of the train is: (A) 20mph_(B) 30 mph (C) 40 mph (D) 50 mph (E) 60 mph If a,b, and ¢ are positive integers, the radicals 4/a+ ® and a4/® are equal when and only when: Wasb=c=1 Banbandenant (e- MD (D)a=b and cis any value (B)a=b and e=a-1 ‘The pairs of values of x and y that are the common solutions of the equations y= (e+ 1% and zy + y = 1 are: (A) 3 real pairs (B) 4 real pairs (C) 4 imaginary pairs (D) 2 real and imaginary pairs (B) 1 real and 2 imaginary pairs In circle 0 chord AB is produced 4, 8 ¢ so that BC equals a radius of the circle. CO is drawn and extended \- to D. AO is drawn. Which of the following expresses the relationship between angles 2 and y? w (B) 2 = 2y D z= (D) there is no special relationship between x and y (E) 2 = 2y or 2 = 3y, depending upon the length of AB 2 THE CONTEST PROBLEM BOOK 45. Given a geometric sequence with the first term 70 and r x0 and fan arithmetic sequence with the first term =0. A third sequence 1,1,2,-++ is formed by adding corresponding terms of the two given sequences. The sum of the first ten terms of the third sequence is: (A) 978 (B) 557 (C) 467 (D) 1068 ‘B) not possible to determine from the informati 46, The graphs of 2 + 3y—6= 0, 42 = 3y — y= F intersect in: (A) 6 points (B) 1 point (C) 2 points (D) no points (@) an unlimited number of points 47. The expressions a + be and (a + iMa+ 6) ae: (A) always equal (B) never equal (C) equal when a+ +e = 1 (D) equal when a+b-+¢=0 (B) equal only when @ = 48. Given triangle ABC with medians AE, BF, CD; FH paraliel and equal in length to AE; BH and HE are drawn; FE extended c H meets BH in @. Which one of the following statements isnot necessarily correct? 5 . (A) BHP is a parallelogram (B) HE = HG (© BH = De . (D) FG = AB (B) FG isa median of ti Boies ile BFH 49. The graphs of y= 24 and y= 2x intersect in: (A) one point whose abaci i 2 (Bone point whowe abscissa i 0 (©) no points () two dstnet points (E) two identical points 50. In order to pass B going 40 mph on a two-lane highway A, going 50 mph, must gain 30 feet. Meantime, C, 210 feet k—- aot ieee fee ee a. If B and C maintain a me € ‘their speeds, then, ——___}— in order to pass safely, A __40meh rust increase his speed by: — 81 (A) 90 mph (B) 10 mphh_ ———}— (©) 5mph (D) 15 mph 1 (B) 3 mph ‘This figure iv not drawn to wcale 10. If 8:2" = Se, then, when y PROBLEMS: 1958 EXAMINATION a 1956 Examination Part 1 The value of + 2(z*) when 2 = 2 is: (10 B16 (CIs (D)36 (Hot ‘Mr. Jones sold two pipes at $1.20 each. Based on the cost his profit on ‘one was 20% and his loss on the other was 20%. On the sale of the pipes, he: (A) broke even (B) lost 44 (C) gained 4¢ (D) lost 10¢ (©) enined 104 ‘The distance light travels in one year is approximately 5,870,000,000,000 miles. The distance light travels in 100 years is: (A) 587 X 10" miles (B) 587 X 10" miles (C) 587 X 10- miles (D) 587 10 miles (E) 587 10-* miles ‘A man has $10,000 to invest. He invests $4,000 at 5 % and $3,500 at 4%. In order to have a yearly income of $500, he must invest the remainder at: (A)6% (B)61% (C)62% (D)63% (HEA A nickel is placed on a table. The number of nickels which ean be placed around it, each tangent to it and to two others is: w+ Bs ©6 Ms Ov . Ina group of cows and chickens, the number of legs was 14 more than ‘ovice the number of heads. The number of cows was: Ws ®7 Oo MR wu ‘The roots of the equation az* + br +.¢ = 0 will be reciprocal if: (and Banke Cena Demb (c= ad Ay-4 B)-3 (Oo M4 (Hs Simplify [YTalV Tal’ the resis: (Aa Bat Ca (Da Ha {A crle of radius 10 inches has ite center a the vertex. C_ ofan equila eral triangle ABC and passes through the other two vertices. The side AC extended through C- intersects the circle at D. ‘The number of degrees of angle ADB is: (A) 15 (B)30 (C) 60 (D) 90 (E) 120 1 L The expreion 1 — . ivviti-vs Wi-VvVB 1 O-Vvi MVE i+ vs equals: 13, 4 1. 16, 17 18, 19. THE CONTEST PROBLEM BOOK If x — 1 is divided by x —1 the quotient is: wr Mt ot mw! @-t Given two positive integers 2 and y with x ar>e (Qe ar but ar <0 (BE) >a" but oF <0 45. A wheel with a rubber tire has an outside diameter of 25 in. When the radius has been decreased a quarter of an inch, the number of revolutions of the wheel in one mile will: (A) be increased about 2% (B) be inereased about 1% (©) be increased about 20% (D) be increased $% (6) remain the same lt: N41 46. Kor the equation 12 — "to be true where N’ is positive, x can have: (A) any positive value less than 1 (B) any value less than 1 (© the value zero only (D) any non-negative value (B) any value 47. An engineer said he could finish a highway section in 3 days with his present supply of certain type of machine. However, with 3 more of these machines the job could be done in 2 days. If the machines all work at the same rate, how many days would it take to do the job with one ‘machine? 6 Be Os (Ws (36 49, THE CONTEST PROBLEM BOOK If p is's positive integer, then 52 +.25 and only if p (A) at least 3 (B) atleast 3 and no more than 35 (C) no more than 35 (D) equal to 35 (E) equal to 3 or 35, can be a positive integer, if Triangle PAB is formed by three tangents to circle 0 and 4 APB=40"; then ‘angle AOB equals: y was" B50 8 55° AN (D) A ow P. st a ). In triangle ABC, CA = CB. On CB square BCDE is constructed away from the triangle. If x is the number of degrees in angle DAB, then (A) 2 depends upon triangle ABC (B) x is independent of the triangle (©) x may equal angle CAD (D) z can never equal angle CAB (E) 2 is greater than 45° but lees than 90° 1957 Examination Part 1 ‘The number of distinct lines representing the altitudes, medians, and i terior angle bisectors of a triangle that is isosceles, but not equilateral, is @9 ®7 Os Ms 3 In the equation 2s" — hr + 2b = 0, the sum of the roots is 4 and the produet of the roots is ~3.'Then A and k have the values, respectively: (A) Sand —6 (B)4and-3 (C) —Band4 (D) -3 and 8 (B) 8 and -3 1 3. The simplest form of 1 — 1 is: 1+ T=3 (Woifaxo (1 Caifex-1 (D) 1 =a with no restriction on a (E)a if ax t 10. a. 12, PROBLEMS: 1957 EXAMINATION 0 ‘The first step in finding the product (Sr +2)(e — 5) by use of the distributive property in the form a(b + ) = ab + ac is: (A) 82 = 132 = 10 (B) 32(x — 5) + B(x — 5) (©) Gr + Dz + r+ 2-3) (D) 3 — 172 - 10 (B82! + Be ~ 15x — 10 ‘Through the use of theorems on logarithms Jog + Jog + tog $ — log ME can be reduced to: . (A) log 4 (BY log (C)1 (D)0 (EB) log St ‘An open box is constructed by starting with a rectangular sheet of metal 10 in, by 14 in. and cutting a square of side x inches from each corner. ‘The resulting projections are folded up and the seams welded. The volume ofthe resulting box is: (A) Or = 482! + det (B) 1400 + 482° + det (© Moe + 248 + 2 (D) 140r — Bket + = (B) none of these ‘The area of a circle insribed in an equilateral triangle is 48". The perimeter of this triangle i (A) 72V5 (BABS (C)36 (D) 24H) 72 ‘The numbers 2, y, 2 are proportional to 2, 3,5. The sum of z, y, and 2 is 100. The number y is given by the equation y = ar ~ 10. Then (a2 (B)3 (C)3 (DS (HDA ‘The value of_z— yh when z= 2 and y = ~2 is: @-8 @B-H Ou (is 26 ‘The graph of y = 2st + 4r +3 has its: (A) lowest point at (—1, 9) (B) lowest point at (1, 1) (©) lowest point at (—1, 1) (D) highest point at (—1, 9) (B) highest point at (—1, 1) ‘The angle formed by the hands of a clock at 2:15 is: (A) 30°" (B) 274° (C) 197}* (D) 1724°(B) none of these Comparing the numbers 10-# and 2-10-* we may say: (A) the frst exceeds the second by 8-10"! (B) the fist exceeds the second by 2-10 (©) the first exceods the second by 8-10-# (D) the second is five times the ist ) the first exceeds the second by 5 50 THE CONTEST PROBLEM BOOK 18 A rational number between Vand Vi is: w 2tV3 @ VV Eis wis mid WM y= VFB + VEFRET, then y is: (2% (B+ (CO M)[x—1]/+/2+1] (B) none of these 15. The table below shows the distance in feet a ball rolls down an in- clined plane in t seconds. CHa aaa o [to {40 {90 | 160 The distance s for (= 25 is: (45 (B25 (C7 (D7 (H) 825 t Part 2 16. Goldfish are sold at 15¢ each. The rectangular coordinate graph showing the cost of 1 to 12 goldfish is: (A) a straight line segment (B) a set of horizontal parallel line segments (C) a setof vertical parallel line segm (D) a finite set of distinet points (E) straight line 17, A cube ie made by soldering twelve 3.inch lengths of wire propery atthe vertices of the cube. If a fly alights at one of the vertices and then walks along the edges, the greatest distance it could travel before coming to any vertex a second time, without retracing any distance, ix (A) 24in.(B) 12in,” (©) 30in, (BD) 1Bin. (8) 36 18. Circle O has diameters ‘AB and CD perpendicular tocach other. Ad is any [> chord intersecting CD at P. O is true only for: (r>1 (B)-1<2<1 Or=02 (D)#=0,2<5-1, 221 220 : where z > means that x can take on all values greater than a and the value equal to a, while 2 < has a corresponding meaning with “tess than.” pee cd ‘The sum of the squares of the first_n positive integers is given by the exrenion MOF OOED i 6 and & are, reece (A) Land? (B)3and5 (C) 2and2 (D) Land 1 (E) 2 and 1 [A regular octagon is to be formed by cutting equal isosceles right tr- angles from the corners of a square. If the square has sides of one unit, the leg ofeach of the triangles as length: w 2tvt @ ia ©! ve -v3 w+ ® ‘The largest of the following integers which divides each of the members of the sequence 161, = 2, 363,02, 8 nyo is: (1 B)eO (C15. (D) 120") 30 If 92+ = 240 + 9%, then the value of = is: (A)01 (B)02 (C)03 (D4 (KE) OS ‘The points that satisfy the system z+ y= 1, x+y < 25, where the symbol ““<"” means “less than,” constitute the following set: (A) only two points (B) an are of a circle (© a straight line segment not including the end-points (D) a straight line sogment including the end-points (©) a single point PROBLEMS: 1087 EXAMINATION 3 35, Side AC of right triangle ABC is . divided into 8 equal parts. Seven line segments parallel to BC are drawn to AB from the points of, division. If BC = 10, then the sum of the lengths of the seven line segments: (A) ean not be found from the given information (B) is38- (C)is 34 (D) i385. (B) is 45. a c Part 3 36, If z+ y = 1, then the largest value of zy is: (&) 1 (B)O5 (©) an irrational number sbout 04 (D) 0.25 (Ho 37, In right triangle ABC, BC AC = 12, and’ AM = 2; MN 4 AC, NP 4 BC;'N is on AB. It y= MN-+ NP, one- half the perimeter of rectangle MCPN, then Weelorm (y= e+ pts 8 14-72 a y Oy- 452 wy-2 P Se @y=B+s * 7 c 38. From a two-digit number NV we subtract the number with the digite re- versed and find that the result is a positive perfect. cube. Then: (A) N’ canmot end in 5 (B) N can end in any digit other than 5 (©) N does not exist (D) there are exactly 7 values for N (B) there are exactly 10 values for NV 39. Two men set out at the same time to walk towards each other from M and N, 72 miles apart, The first man walks at the rate of 4 mph. ‘The second man walks 2 miles the frst hour, 2} miles the second hour, 3 miles the third hour, and so on in arithmetic progression. Then the men will meet: (A) in7 hours (B) in 8 hours (©) nearer M than NV (D) nearer N than M(B) midway between M and a THE CONTEST PROBLEM BOOK 40. If the parabola y = —2* + bx — 8 has its vertex on the z-axis, then must be: (A) & positive integer (B) & positive or a negative rational number (C) & positive rational number (D) & positive or a negative irrational number (B) a negative irrational number ng OF + (0 Dy 41, Given the system of equations (FI), ay, the following values of @ is there no solution for = andy? 1 Bo ©-1 m2 wavs For which one of 42. S= i +i, where i= =i and n is an integer, then the total number of possible distinet values for S 1 2 ©3 (D) 4 (E) more than 4 43, We define a lattice point as a point whose coordinates are integers, ero ‘admitted. ‘Then the number of lattice points on the boundary and inside the region bounded by the z-axis, the line z= 4, and the parabola y= is: (A)24 (B) 35. (©) 34 (D) 30 (E) not finite 44, In wiangle ABC, AC = CD ° and CAB — %ABC = 30° ‘Then %BAD is: (A) 30° (B) 20° (C) 2 (D) 10° (E) 18° a 8 45, If two real numbers 2 and y satisfy the equation 2/y = z — y, then: (A) 224 and r<0 where za means that x can take fany value greater than a or equal to @ (B) y can equal 1 (©) both x and y must be irrational (D) = andy eannot both be integers (B) both 2 and y must be rational 46, Two perpendicular chords intersect in a circle. The segments of one chord are 3 and 4; the segments of the other are 6 and 2, Then the diameter of the ciele is - _ AVVO BVH OV DVB OVE PROBLEMS: 1057 EXAMINATION 38 47. In circle 0, the midpoint of radius OX is Q; at Q, ABL XY. The semicircle with AB as diameter intersects XY in M. Line AMf itersects circle O in C, and line BM interseets circle O in D. Line AD is drawn. Then, if the radius Of circle O is 1, AD is: x| (A rv2_ (Br (©) nota side of an inseribed regular polygon oY oni 48, Let ABC be an equilateral triangle ingeribed in cirele O. M isa point = Gea ‘on are BC. Lines AM, BM, and CM are drawn, Then AM (A) equal to BH + CH (B) less than BM + CHL (C) greater than BM + CM (D) equal, less than, or greater than BM + CM, depending upon the positon of * (B) none of these 49. The parallel sides of a trapezoid are 3 and 9. ‘The non-parallel sides are 4 and 6. A line 7 parallel to the bases divides the trapezoid into two trapezoids of equal perimeters. ‘The ratio in which each of the non-parallel sides is divided is: 50. In circle O, Gis a moving point on diameter AB. AA’ is drawn per- pendiculer to AB and equal to AG. BB’ is drawn perpendicular to 4B, on the same side of diameter AB ax AA’, and equal to BG. et 0" be the midpoint of ’B'. Then, as G moves from A to B, point 0' (A) moves on a straight line parallel to AB (B) remains stationary (©) moves on a straight line perpendicular to AB (D) moves in a smal cirle intersecting the given circle () folloms a path which is nether a circle nor a straight line 6 THE CONTEST PROBLEM BOOK 1958 Examination Part 1 1. The value of (2 — 3(2 — 3) is: @5 @)-5 © ©) au t—2}, wan # sae: ()y-2 (B)z-y © wy 3. OF the following expressions the one equal to wo £6 o£ © ge 4, In the expression 2+ 1 exprewion, evaluated for 2 = }, equals: @s @-3 ©1 O)-1 oe 8 Theerpreaion 2+ V3+ 515 + Gt 2 B2-v2 O2+v2 M22 HM equals: wv 6. The arithmatie mean teen ES nd (the symbol » means “not equal to” (A)2 if @ #0 (BY (C)1, only when 70, is a=0 Wja/z (Bz 7. A straight Line joins the points (—1, 1) and (8,9). Tts z-intercept is 2 2 wo 2 ©7 m2 ws 8. Which of these four numbers VP, V3, D016, W/=1--/(09)%, is (are) rational: (A) none (B) all (C) the first and fourth —(D) only the fourth (E) only the first 9. A vale of = sisyng the equation ot 5 = (6 — 2 i w Pte @ tet @tet wmszt = © 10. For what real values of &, other than & = 0, does the equation 2+ ke + = 0 have real roots? (The symbol z >a means that x can take on all values greater than a and the value @ itself; x O (R21 (hall values otk (B) no values of 11. The number of roots satisfying the equation V5—z = 253 is: (A) unlimited (B)3-(C)2 (D1 HO are rE then meals: . (og (v a @ Tog (1+ #) B) bee rE (©) one (D) we + gH) (B) SBE 13, The sum of tmo numbers is 10; their product ie 20. ‘The sum of their reciprocal is w @} 1 M2 ws 14, At a dance party a group of boys and girls exchange dances as follows: ‘one boy dances with 5 girls, a second boy dances with 6 girls, and so on, the last boy dancing with all the girls. If b represents the number of boys and g the number of girls, then: Ab=g B)b=9/5 Cb=9-4 W)b=9 (B) It is impoasible to determine a relation between b and g without knowing the total number of boys and girls. 15. A quadrilateral is inzeribed in a circle. If an angle is inscribed into each of the four segments outside the quadrilateral, the sum of these four angles, expressed in degrees, is: (A) 1080 (B) 900 (C) 720 (D) 540 (E) 360 16. The area of a circle inseribed in a regular hexagon is 100r. The area of the hexagon is: (A) 600 (B) 300 (©) 20072 (D) OVE (B) 1075 17. If & is positive and log 2 > log 2 + # log z, then: (A) 2 bas no minimum or maximum value (B) the maximum value of zis 1 (C) the minimum value of 2 is 1 (D) the maximum value of x is 4 (B) the minimum value of = is 4 18, The area of a circle is doubled when its radius + is increased by n. Then r equals: AmVv2+1 BavF-1) On (D)n2@- vB ) Vari a1 THE CONTEST PROBLEM BOOK ‘The sides of a right triangle are a and b and the hypotenuse is «A perpendicular from the vertex divides ¢ into segments r and s, ad- jncent respectively to a and 6, If a:b = 1:3, then the ratio of r to @L3 LI Cis O30 Hv If 42 — at = 24, then (22)* equals (A)5V5 (B) VE (C)25V5_ (D) 125 (B) 2 In the accompanying figure £ CE and DE are equal chords of a circle with center 0. Aro AB is a quarter-ircle. ‘Then the ratio of the area of fl 7 , triangle CED to the area ge to aX (A) V2:1(B) 3:1. (©) 4:1 (D) 3:1 2 => (B) 2:1 2. A particle is placed on the parabola y = 2*—2—6 ata point P ‘whose ordinate is 6. It is allowed to roll along the parabola until it reaches the nearest point Q whose ordinate is —6. The horizontal distance traveled by the particle (the numerical value of the difference in the abscissas of P and Q) i @5 B4 C3 2 H1 . If, in the expresion 2 — 3, z increases or decreases by a positive ‘amount, the expression changes by an amount: (A) #2ar $a? (B) Qar tat (C) kat — 3 (D) (eta) 3 @®@-a-3 ‘Aman travels m feet due north at 2 minules per mile. He returns due south to his starting point at 2 miles per minute. The average rate in miles per hour for the entire trip is: (A)75 (B)48 (C45. (D) 24 (B) impossible to determine without knowing the value of m 1 logs elogs k= 8, then 2 equals; (HE BSH (CE (D) 2B (H) 125 ‘Avset of m numbers has the sum s. Each number of the set is increased by 20, then multiplied by 5, and then decreased by 20. The sum of the numbers in the new set thus obtained is: (A)s+ 200 (B) 58+ 80m (C)s (D)Ss (E) Ss + 4m 31 Ea PROBLEMS: 1058 EXAMINATION # ‘The points (2, —3), (4,3) and (6, 4/2) are on the same straight line. ‘The value(s) of k is (are): (A) 12° (B)=12, (©) 412 (D) 120r6 (H) 6or A.16-quart radiator is filled with water. Four quarts are removed and re- placed with pure antifreeze liquid. Then four quarts of the mixture are removed and replaced with pure antifreeze. This is done a third and a fourth time, The fractional part of the final mixture that is water is: pee OL gee rere Oreos OF oF w% In a general triangle ADE (as shown) € lines BB and EC’ are drawn. Which of the following angle rela- tions is true? A rte=atb Byt+e=a+d © mtz=wtn 3 @Ortitnewtetm A—H ¢ ° © rt+ytasatbtm a ; HCFW Bet Od—+2 Moo+2) HL+m 1 If ay = and 4 + 4, then (2 + y)* equals: ‘The altitude drawn to the base of an isosceles triangle is 8, and the per- imeter is 32. The area of the triangle is: (A) 56 (B) 48. (C)40 (D2 (E) With $1000 a rancher is to buy steers at $25 euch and cows at $26 each. If the number of steers «and the number of cows ¢ are both positive integers, then: (A) this problem has no solution (B) there are two solutions with s exceeding © (C) there are two solutions with ¢ exceeding « (D) there is one solution with ¢ exceeding ¢ (B) there is one solution with ¢ exceeding = . For one root of xt + bz +.¢ = 0 to be double the other, the coefficients 4, b, © must be related as follows: (A) 40 = 9¢ (B) 28° = Gee (C) 2 = 9a (D) HF — Bac = 0 (B) 9b = 2ae ‘The numerator of a fraction is Gr + 1, the denominator is 7 — 42, and z can have any value between —2 and 2, both included. The values of 2 for which the numerator is greater than the denominator are: WiI. Then a: equals: 8) (a (10 () -17 Part 3 ‘The roots of Azt + Be + C= 0 are r ands. For the roots of Btpete=0 tobe # and «, p must equal: w = aac ®) sa aac FF (p20 - BF 2, 48, 46, a. PROBLEMS: 1958 EXAMINATION a In acircle with center O chord AB = chord AC. Chord AD cute BC in B. If AC = 12 and AB = 8, then AD equals: (A272 (C/A (D2 (B) 18 AB isthe hypotenuse of aright triangle ABC. Median AD = 7 and median BE = 4, The length of AB is: (10 B)5VF_C)5V2 (D)2VB_ (B) 2V5 Given the true statements: (1) If @ is greater than b, then ¢ is greater than d (2) If ¢ isless than d, then ¢ is greater than j. A valid con- clusion is: (A) If @ istess than b, then ¢ is greater than f (B) If ¢ is greater than f, then a is less than 5 (© If isless than f, then a is greater than & (D) If a isgreater than b, then ¢ is leas than f (B) none of these Accheck is written for x dollars and y cents, x and y both two-digit ‘numbers. In error it is exshed for y dollars and z cents, the incorrect amount exceeding the correct. amount by $17.82. Then: (A) = cannot exceed 70 (B) y can equal 2r (© the amount of the check eannot be a multiple of 5 (D) the incorrect amount ean equal twice the correct amount (E) the sum of the digits of the correct: amount is divisible by 9 For values of less than 1 but greater than —4, the expression Bnet? m2 has: (A) no maximum oF minimum value (B) a minimum value of +1 (© a maximum value of +1 (D) a minimum value of —1 (© a maximum value of ~1 ABCD is. rectangle (see the ‘accompanying diagram) with P barsieg: any point on AB. PS 1. BD =e and PR1LAC. AF BD of and PQ 1 AF. ‘Then PH + PS is equal to: Gi, Oe rN ( 4B (© Pry ar (D) 4F (&) EF 49. THE CONTEST PROBLEM BOOK ith center 0 is 10 unite. C’ is a point 4 unite from 4, andon AB. D isa point 4 units from B, and on AB. P is ‘any point on the circle. Then the broken-line path from C to P to D (A) has the same length forall positions of P (B) exceeds 10 units for all positions of P (©) cannot exceed 10 units (D) is shortest when CPD isa right triangle (B) is longest when P is equidistant from C and D. In the expansion of (a + 0)* there are n+ 1 dissimilar terms. The number of dissimilar terms in the expansion of (a + b+ 0)" is (ATL B)33 (C55 (D) 6G (B) 132 ). In this diagram a scheme is indicated for 9} ape tscoiting ll the points of eegment, AB tt ‘ wrth thet of segment 4°B" ad reciprocally To describe the sociation heme anal cally Ie x be the tance from s point Pon AB to D tnd lt y be te distance rom ihe sociated point of 4'B" to D Then for any pair of exvocatd point, Weve stp oqo (a) tea () ta — a1 (18 tp aa © 0 1959 Examination Part Each edge of a cube is increased by 50%. The percent of inerease in the surface arce of the eube is: (A) 50 (B) 125 (©) 150 (D) 300 (E) 750 ‘Through & point P inside the triangle ABC a line is drawn parallel to the base AB, dividing the triangle into two equal areas. If the altitude to AB has a length of 1, then the distance from P to AB is Ww, BL @2-v2 MAF wit? PROBLEMS: 1059 EXAMINATION 6 3. If the diagonals of a quadrilateral are perpendicular to each other, the figure would always be included under the general classificato (A) rhombus (B) rectangle (C) square (D) isosceles trapezoid (B) none of these 4. If 78 is divided into three parts which are proportional to 1, 4, 4, the riddle part is: (A) 9} (B13. (CITE (D) 18} BH 5. The value of (256)"-(256)o" is (44 (B) 16 (©) 64 (D) 256.25 (B) -16 en the true statement: If a quadrilateral is a square, then it is a rectangle. It follows that, of the converse and the inverse of this true statement: (A) only the converse is true (B) only the inverse is true (©) both are true (D) neither is true (B) the inverse is true, but the converse is sometimes true 7. The sides of a right triangle are a, a +d, and a + 24, with a and 4 both positive. The ratio of @ to d is: HES But O21 M)s1 (34 8. The value of 24 — Gx + 13 can never be less than: 4 B45 (5 (7 (HB 9. A farmer divides his herd of 1 cows among his four sons so that one ‘son gets one-half the herd, a second son, one-fourth, a third son, one-fifth, and the fourth son, 7 cows. Then m is (A) 80 (B) 100" (CG) 140 (D) 180 (E) 240 10. In triangle ABC, with AB = AC = 36, a point D is taken on AB ata distance 1.2 from A. Point D is joined to point # in the pro- longation of AC so that triangle ABD is equal in area to triangle ABC Then AB equals: (48 B54 O72 (10s 26 11, The logarithm of .0625 to the base 2 is: (A) 0% (B).25 (C)5 (D) 4 (H) -2 12, By adding the same constant to each of 20, 50, 100 « geometric progression results. The common ratio is: 5 4 3 1 we MF OF 3 13. The arithmetic mean (average) of a set of 50 numbers is 38. If two num- bers, namely, 45 and 55, are discarded, the mean of the remaining set of ‘numbers is: (A) 365 (B)37 (GC) 372 (D375 (H) 37.82 1 om; ©} 4 15. 16, 17 18, 19. a THE CONTEST PROBLEM BOOK Given the set $ whose elements are zero and the even integers, positive fand negative. Of the five operstions applied to any pair of elements: (1) addition (2) subtraction (3) multiplication (4) division (6) finding the arithmetic mean (average), those operations that yield only elements of 8 are: Call (B)1,2,3,4 (1,235 (D123 1,35 In a right triangle the square of the hypotenuse is equal to twice the product of the legs. One of the acute angles of the triangle is: (a) 15" (B) 30° (©) 45° (D) 60” (B) 75° be +4 fomse et : ZaMER FA SA hen simplified is (=NG-8) (tts Gert! wees OH OR, om me ‘The expression where a and b are constants, and if y = 1 when 2-1, and y= 5 when x= —5, then a+b equale: @-1) BO 1 Mw (Hu ‘The arithmetic mean (average) of the first n positive integers 2 we On (wt + (eet z With the use of three different weights, namely, 1 1b., 3 Ib., and 9 Ib, how many objects of different weights can be weighed, if the objects to be weighed and the given weights may be placed in either pan of the scale? Wb GBB Ou Me «7 tis given that varies directly as y and inversely as the square of 2, fand that z= 10 when y= 4 and z= 14. Then, when y= 16 and z= 7, 2 equals: (A) 180 (B) 160 (C) 154 (D) 140 (E) 120 Part 2 If p is the perimeter of an equilateral triangle inscribed in a circle, the area of the ciele ia: ww we wa we wv wot oF OFF me om? 2. The line joining the midpoints of the diagonals ofa trapezoid hes length 3 If the longer base is 97, then the shorter base is: (a) 9 (B) 92 (C)9L_(Dy90 CE) 89 a PROBLEMS: 1059 EXAMINATION ‘The set of solutions for the equation logis (a? — 15a) = 2 co (A) two integers (B) one integer and one fraction (C) two irrational numbers (D) two non-real numbers (B) no numbers, that is, the set is empty. ‘A chemist has m ounces of salt water that is m% salt. How many ‘ounces of salt must he add to make a solution that is 2m% salt? ” 2m m m “orm ® wm © te am +2 +) 2m Oma ‘The symbol |a| means +a if a is greater than or equal to zero, and =a if @ is lesa than or equal to zero; the symbol < means “less than’; the symbol > means “greater than, ‘The set of values 2 satisfying the inequality |3 — 2| <4 consiate of all x such that: We<49 B)e>1 (C1<#<49 D)-1 means “greater than or equal to"; the symbol < means “less than oF equal to” In the equation (x — mt — (x ~ n) = (m — nit, m isa fixed posi- tive number. and n is a fired negative number. The set of values satisfying the equation (x20 (B)x Sn (C)x= 0 (D) the set of all real numbers (B) none of these ‘The base of triangle i 80, and one of the base angles is 60°. The sumn of the lengths of the other two sides is 90, The shortest side is: (45 (B)40 (C36 (D) THB ‘When simplified the product (1 — 41 — (1 = 8) ++: (1 = I/n) becomes: 1 2 An = 1) 2 . wt m2 © Ow © eet eee ae 1) If 42+ V% = 1, then (A) is an integer (B) is fractional (C) is irrational (D) is imaginary (B) may have two different values 30, 4 48, 45 PROBLEMS: 1959 EXAMINATION or Let S be the sum of the first nine terms of the sequence, rHa e+ 2a A+ 30, ‘Then $ equals: | (ay ate + voi (DEF + 0 In triangle ABC, BD is a median. CF intersects BD at E so that BE = ED. Point F ison AB. Then,it BP = 5, BA equals (A) 10 (B) 12° (C15 (D) 2 (E) none of these Part 3 On the same side of a straight fine three citcles are drawn as follows: & circle with a radius of 4 inches is tangent to the line, the other two cireles are equal, and each is tangent to the line and to the other two circl ‘The radius of the equal circles is: (A) 2 (B)20- (C18 (D) 16 (1B Given three positive integers a, b, and c. Their greatest common divisor is D; their least common multiple is M. Then, which two of the following statements are true? G) the product MD cannot be less than abe (2) the product MD cannot be greater than abe (@) MD equals abe if and only if a,b, are each prime (4) MD equals abe if and only if a, b ¢ are relatively prime in pairs (This means: no two have # common factor greater than 1.) @1L2 B13 O14s (DAI HAs ‘The sides of a triangle are 25, 39, and 40. The diameter of the cireum- scribed circle is 125 a8 HB (oe Dy (B40 ‘The roots of 2 + br + ¢ = 0 are both real and greater than 1. Let e=btetl. Then & (A) may be less than zero (B) may be equal to zero (C) must be greater than zero (D) must be less than zero (E) must be between —1 and +1. WE ogee. 2 gu) = oes tan yu wo! we Ww wT wa 47, 49. Given the THE CONTEST PROBLEM BOOK A student on vacation for d days observed that (1) it rained 7 times, morning oF afternoon (2) when it rained in the afternoon, it was clear in the morning (3) there were five clear afternoons (4) there were six clear momings. Then d_ equals: @7 Bs Ow Mn Hw Assume thatthe following three statements are true: 1. Allfreshmen are human. II All students are human. II. Some students think Given the following four statements: (1) All freshmen are students. (2) Some humans think. (3) No freshmen think. (4) Some humans who think are not students. Those which are logical consequences of I, I, and 111 are 2 Bs O2%3 (D4 E12 ynomial exch + axe! ob +> baat + Oy where 4 postive integer or 2er, anda» is & positive integer. The remaining a's are integers o zero. Seth = n+ e+ |au|+ [an|-+ o> + | [See example 25 for the meaning of |x|.) The number of polynomials with h='3 i 3 B5 ©6 M7 Ho For the infinite series 1 — 3 — B+ $= te — He tee — rhe o> det 'S be the (limiting) sum. Then S equals: Mo @? @! MZ « a 2 A.club with 2 members is organized into four committees in accordance ‘with these toro rules: (1) Each member belongs to two and only two committees. (2) Bach pair of committees has one and only one member in common. Then 2: (A) cannot be determined (B) has a single value between 8 and 16 (C) hhas two values between 8 and 16 (D) has a single value between 4 and & (B) has two values between 4 and 8 1960 Examination Part 1 If 2 isa solution (root) of 2" + Ar + 10 = 0, then A equals: 10 BS ©2 D)-2 @-9 10, 1. The fraction % PROBLEMS: 1960 EXAM: ATIO It takes 5 seconds for a clock to strike 6 o'clock beginning at 6:00 o'clock precisely. IF the strikings are uniformly spaced, how long, in seconds, does it take to strike 12 o'clock? (A) 9 (B)10 (C) 1 (D) 148 (B) none of these Applied to a bill for $10,000 the difference between a discount of 40% land two successive discounts of 96% and 4%, expressed in dollars, is: (AO (B) 44 (©) 256 (D) 400 (H) 416 ach of two angles of a triangle is 60° and the included side is 4 inches. ‘The area of the triangle, in square inches, is: (Asvi (BB (C)4VE (D)4 | 23 ‘The number of distinet points common to the graphs of z+ yf = 9 and y= 9 is (A) infinitely many (B) four (C) two (D) one (E) none . ‘The circumference of a circle is 100 inches. The side of a square inscribed in tia eile, expressed in inches, x (A) V2 (By BYE (10 (py OV? soy Circle I passes through the center of, and is tangent to, circle II. The area of circle I is 4 square inches. Then the ares of eile II, in square inches, (8 (B)8V2 (C)8VF (D) 16 (E) 16VE . The number 2.5252525 --- can be written as a fraction. When reduced to lowest terms the sum of the numerator and denominator of this frac- tion is: (A)T (B) 29 (C) 141 (D) 349 CE) none of thebe a+ he + 2b i FSP (with suitable restrictions om the values of a, b, and ¢) (A) irreducible (B) reducible to —1 anbte (©) reducible toa polynomial of three terms (D) reducible to apbqe atb-e (B) reducible to SESE Given the following six statements: (1) All women are good drivers (2) Some women are good drivers (B) No men are good drivers (4) All men are bad drivers (8) Atleast one man is a bad driver (@) All men are good drivers ‘The statement that negates statement (6) is AM B® OO Ow Ow 13, i 1. 16. 1". 18, THE CONTEST PROBLEM OOK |. For a given value of & the product of the roots of Ske + 2-150 is 7. The roots may be characterized as: (A) integral and positive — (B) integral and negative (©) rational, but not integral (D) irrational (E) imaginary ‘The locus of the centersof all circles of given radius a, in the same plane, passing through a fixed point, is: (A) 8 point (B) a straight (B) two circles ‘The polygon(s) formed by y = 32+ 2, y= —Br +2, and y= 2, is (ae): (A) an equilateral triangle (B) an isosceles triangle (C) a right triangle (D) a triangle and a trapezoid (B) a quadrilateral (©) two straight lines (D) a circle If a and b are real numbers, the equation Br — 5 +a = br +1 has ‘9 unique solution + [the symbol a ¥ 0 means that a is different from ero]: (A)for all @ and b (B)if a2 (C)ifax6 (Dif bwO Bit ows ‘Triangle I is equilateral with side A, perimeter P, area K, and cir- cumradius R_ (radius of the circumscribed circle). Triangle II is equi- lateral with side a, perimeter p, area k, and cireumradius r. If A is different from a, then: (A) Pip = Rir only sometimes (B) P:p = Rir always (© Pip = K:k only sometimes (D) R:r = Kzk always (E) Rir = Kzk only sometimes In the numeration system with base 5, counting is as follows: 1, 2,3, 4, 10, 11, 12, 18, 14, 20, --- The number whose description in the decimal system is 69, when deseribed in the base 5 system, is « number with: (A) two consecutive digits (B) two non-consecutive digits (©) three consecutive digits (D) three non-consecutive digits (©) four digits ‘The formula N = 8-102" gives, for certain group, the number of individuals whose income exceeds dollars, The lowest income, in dollars, of the wealthiest 800 individuals is at least (A) 10 (B) 10 (C) 10" (D) 10* (E10 ‘The pair of equations 3+” = 81 and 81: has: (A) no common solution (B) the solution 2 = 2, y= 2 (©) the solution x = 24, y= 1 (D) 8 common solution in positive and negative integers (B) none of these 19. m1. m4, PROBLEMS: 1960 EXAMINATION m Consider equation 1: e+ y-+ 2 = 46 where 2, y, and = are positive integer, and equation I: + y+ 2-4 w= 40, where 2 y, 2 and tare podive integers, Then (A) T can be solved in consecutive integers (B) ean be solved in consecutive even integers (©) Ti can be ave in consecutive integare (D) II can be solved in consecutive even integers (E) II can be solved in consecutive odd integers. The coefiient of 2° inthe expansion of (5 — 2) ix (A) 56 (B) 56 (C)14 (D) -14 (ENO Part 2 ‘The diagonsl of square I is a +b. The perimeter of square II with twice the area of I is (A (a+ dF (B) V20+ HF (C)2a+b) (D) VBa+d) ©) 4a +) ‘The equality (e+ m)t ~ (x +m) unequal non-zero constants, is satisfied by 2 (A) @ = 0, b hase unique non-zero value (B) a= 0, 8 has two non-zero values (© 6 =0, a hasa unique non-zero value (D) 6 = 0, a has two non-zero values (B) a and b cach have @ unique non-zero value. ‘The radius R of a cylindrical box is 8 inches, the height ‘The volume V amount when R jis increased by 1 inches as when His increased by 2 inches. This condition is satisfied by: (A) no real value of (B) one integral value of = (C) one rational, but not integral, value of x (D) one irrational value of (B) two real values of 2 = (m — mt, where m and n are im + bn where: If logss216 = x, where x is real, then = (A) A non-square, non-cube integer (B) a non-square, non-cube, non-integral rational number (©) an irrational number (D) & perfect square (B) a perfect cube Let m and n be any two odd numbers, with nm less than m. ‘The largest integer which divides all possible nurnbers of the form mt — n? is: W2 G4 O©6 Ms Hie n 28, a. 32. | Given the line 32 + 5y = 15 and a point on th THE CONTEST PROBLEM BOOK se <2. (The symbol |a| means +a if @ is positive, ~a if a is negative, Oif 4 i zero, The notation 1 < a <2 means that a can have any value between I and 2, excluding 1 and 2.) @Miu ® [zi <6 Let S be the sum of the interior angles of « polygon P for which each interior angle is 74 times the exterior angle at the same vertex. Then Find the sot of r-values satisfying the inequality | as TP may be regular @ s P ist regular 8 P is regular (0) s P is not regular @s may or may not be regular i z ‘The equation x — 2, = 3 - 1 has (A) infinitely many integral roots (B) no root (C) one integral root (D) two equal integral roots (E) two equal non-integral roots Five times A's money added to B's money is more than $51.00. Three ‘times A’s money minus B's money is $21.00. If a represents A’s money in dollars and b represents B's money in dollars, then: a>%b>6 Bar%b<6 Card, bab (D) a> 8, but we ean put no bounds on 6 (B) 2a = 3b line equidistant from the coordinate axes. Such a point exists in: (A) none of the quadrants (B) quadrant [only (C) quadrants I, IL only (D) quadrants I, 1, IH only (B) each of the quadrants Part 3 For #4245 wbeafactor of 2+ mt +, the values of p and ‘ql be, respectively: (A) 2,5" (B)5,25 (©) 10,20 (D) 6,25 (B) 14,25, In this figure the center of the cirele is 0. AB 1 BC, ADOE isa straight line, AP = AD, and AB has length ‘ice the radius. Then 4 (A) AP = PB-AB (B) AP-DO = PB-A (©) 4B = 3D-DE (D) 4B-AD = 08-20 (B) none of these PROBLEMS: 1060 EXAMINATION . ‘You are given a sequence of 58 terms; each term has the form P -+ n where P stands for the product 2-3-5- «++ -61 of all prime numbers (@ prime number is a number divisible only by 1 and itself lese than for equal to 61, and n takes, successively, the values 2, 3, 4, +++, 59, Let N’ be the number of primes appearing in this sequence. ‘Then NV Wo G6 Or WT (ss ‘Two swimmers, at opposite ends of a 90-foot pool, start to swim the length of the pool, one at the rate of 3 feet per second, the other at 2 feet per second. They swim back and forth for 12 minutes. Allowing no loss of time at the turns, find the number of times they pass each other. (a2 B21) (D) 19H) 18 From point. P outside a cirele, with a circumference of 10 units, « tangent ia drawn. Also from P a secant is drawn dividing the circle into unequal fares with lengths m and n, It is found that the length of the tan- gent, is the mean proportional between m and n. If m and ¢ are integers, then 1 may have the following number of values: (A) zero (B) one (C) two (D) three (E) infinitely many Let t1, a2, a be the respective sums of , 2n, 3n terme of the same arithmetic progression with a as the first term and d as the common difference. Let R = s: — 8 ~ 81. Then R is dependent on: (Aya and d (B)d and.» (C)a and n (D)a, dy and (B) neither @ nor d norm ‘The baso of a triangle is of length , and the altitode is of length A. A rectangle of height x is inscribed in the triangle with the base of the rectangle inthe base ofthe triangle. The area ofthe rectangle is Ea 2 Hb» (Eh 2) (D)2(b— 2) (E) 2(h — 2) In this diagram AB and AC are the equal sides of an isosceles triangle ABC, in which is inseribed equilateral tri- angle DEF. Designate angle BFD by a, angle ADE by b, and angle FEC by ¢. Then: ete (ayo=2F* (By bme 2 bre (a= 3 (B) none of these (D)a= 4 THE CONTEST PROBLEM BOOK ard 39, To satisfy the equation * +? = —? az Hor 2 and b must be (A) both rational (B) both real but not rational (C) both not real (D) one real, one not real (FE) one real, one not real or both not real 40, Given right triangle ABC with legs BO = 3, AC = 4. Find the length Of the shorter angle titer from to the hypotenuse: (a BYB= cp) BVE= 8 yeas (SV 1 w o 25 we II Answer Keys AaRopOrmuam> BESBRNER Pore meD SSSSSRE8 Paur dao III Solutions 1950 Solutions La De + az + Gr = 64; 216 = & = 45 6. Solve the second equation for z, substitute into the first equation and simplify: a= ¥t3 2(-¥43) +0(- +3)4 yt 1-0; s¥ + Wy -7 = 0. 7. Placing 1 as indicated shifts the given digits to the left, so that ¢ is now the hundreds’ digit and u is now the tens’ digit 2 100! + 10u + 1 ” 10. LL. 12, 13, M4. 1B. 16. THE CONTEST PROBLEM BOOK Let 1 be the original radius; then 2r = now radius, x7 = original area, 4x7? = new area det =n a +100 = 300, Percent increase in area = (Of all triangles inscribable in a semi-circle, with the diameter as base, ‘the one with the greatest area is the one with the largest altitude (the radius); that is, the isosceles triangle. Thus the area is }-2rer = #3 PR rele Ap eA usta v3 VEtV2 346 ” Ream ~ Bate denominator smaller and, consequently, the fraction C_ larger. Therefore an increase inn makes the It is theorem in elementary geometry that the sum of the exterior tangles of any (convex) polygon is a constant, namely two straight angles. Set each of the factors equal to zero: 220, 2-420, #84 2% (2 2(z-1) = 0; 220,421 Geometrically, the problem represents a pair of parallel lines, with slope equal to 4, and hence there is no intersection point. Algebraically, if oz + by =e and a+ by =e, then, multiplying the first equation by bs, the second by b,, and sub- tracting, we have cide — eb aby aay (a,ba — asbidz = by — cabs Thus if ayby — ashi = 0 and c, x0, ¢ #0, 2 is undefined and no solution exis It is understood that we restrict ourselves to rational coefficients and integral powers of . With this restriction z* + 4 is not factorable. Part 2 ‘The binomial expansion of (z+ y)* has n-F 1 terms, Thus (a + ab)(a = 38)F = (at — 9by¢ hhas 4+ 1 = 5 terms a. 18. 19. a. . By ordinary } Let R be the yearly rent. Then SOLUTION: 950 EXAMINATION a ‘To obtain a formule directly from the table involves work beyond the indicated scope. Consequently, the answer must be found by testing the choices offered. (A) is immediately eliminated since, for equally spaced ‘values of 2, the values of y are not equally spaced; .y cannot be a linoar funetion of 2. (B) feils beyond the second eet of values. (C) satisfies the entire table. (D) fails with the first set of values. OF the four distributive laws given in statements (1), (2), (3) and (5), only (1) holds in the real number field. Statement (4) ia not to be con- fused with log (2/y) = log ~ log y. +. (B) is the correct choice. The job requires md man-days, 0 that one man can do the job in md days. Therefore, (m-+r) men can do the job in md/(m +r) days; m oa mee mee —in this instance long and tedious operation— the answer (D) is found to be correct. By the Remainder Theorem, we have R= I" +1 = 2 Note: The use of aynthetic division shorten the work considerably, but synthetic division itself is generally regarded tas beyond the indicated scope of this examination. The Remainder ‘Theorem is likewise beyond the scope of this examination. ‘= 12, bw =8 and to = 6, Eliminating h we obtain 1 = 3u/2. Eliminating |, 3u'/2= 6. w= 2, 1=3, h=4, and V = Wh = 24; V2 m= (Woh)? = Dhshw-bo = 12-841 PO Va 46 = . Let P be the original price. Then a discount of 10% gives a new price P= .1P = SP. Following this by a discount of 20%, we have OP ~ 2(9P) = .72P. ‘Thus the net discount is P —.72P = .28P or 28%; Ot Obst. net discount = dy + ds ~ did = ty + oy — gg = 28% 106 5h 10,000 = r — 2R — 325; 10,000 =k - He - 325; R +R = $1000 and £ ~ $83.33. 2 THE CONTEST PROBLEM BOOK a vi dom 2-25 16-8rta, O= F-92418 = HE - 3; A check is obtained only with x = 3, aso (58) mae = tons = 8 26. b = logiom + login = logiemn; mn = 10; m= 10/r Hogiom = logiel0* — logion = logie(10*/n); mm = 108/n. 27. If a car travels a distance dat rater and returns the same distance atrate ry, then total distance 2d Dre average speed = tal time n+ dn an 2-30-40 _ 240 eo 28 2 a4 mph, 28. Notice that A and B both travel for the same amount of time. Since A travels 48 miles while B travels 72 miles, we have raed 29, Since the first machine addresses 500 envelopes in 8 minutes, it addresses '500/8. envelopes in 1 minute and 2-500/8 in 2 minutes. If the second ‘machine addresses 500 envelopes in z minutes, then it addresses 2-500/2 envelopes in 2 minutes. To determine z 40 that both machines together ‘address 500 envelopes in 2 minutes we use the condition 2 100 = 500 op 24 ay op Ly tal 2 1 g° + ao-iat z a 30. If B is the original number of boys and G the orginal numberof gis then 2G = 40, o-B BI. Let 2 be the number of pairs of blue socks, and let_y be the price of a pair of blue socks. Then 2y is the price of a pair of black socks and + dey = H4-2y + ry). Divide by y and solve for x “die = 4216 = 1:4, 2 = 16; 2, 33. cm 35. . Let C= merchant's cost, L |. This sequence is an infinite geometric progression with SOLUTIONS: 1950 EXAMINATION 88 ‘Before sliding, the situation is represented by a right triangle with hypo- tenuse 25, and horizontal arm 7, 60 that the vertical arm is 24. After sliding, the situation is represented by a right triangle with hypotenuse 25, and vertical arm 20, s0 that the horizontal arm is 15. 15 — 7 = 8 so ‘that (D) is the correct choice. ‘The crocs section of the pipe is. xR, large pipe/small pipe = 6/1" =36, ‘The large pipe has the carrying eapacity of 36 small pipes. ince C = 2er, letting rs be the original radius, ry the final radius, we Have 25 = 20 2ery— ary = Be(ry— 2); vine ra = 5/2 For any right triangle we have anrtb—r where ris the radius of the inscribed circle. W®=atb—e 2+ 10 ~ 28 = 8; =4 price, M = marked price, 'S = selling price, and P = profit. Then Reference to the graph of y = logat, a > 1, or to the equality x = a¥, shows that (A), (B), (C), and (D) are all correct. fara] [22 |-m2- 0 “1d ‘Thus if a ists fist term and # its sum, we have Hence (4) and (5) are correct statements, oy THE CONTEST PROBLEM BOOK go. 2 =I @tDG-) aS $s 241, forall 2 #1; the limit of 2 +1 as 2—+1 is 2, 60 that (D) is the correet choice. ‘Note: This problem is beyond the scope of the contest, 41. The graph of the function y = az* + br +c, a 0, is a parabola with its axis of symmetry (the line fV extended) parallel to the y-axis, ¥, ¥ y| pw Jo of Wo ek v We wish to find the coordinates of V. Let y= & be any line parallel to the z-axis which intersects the parabola at two points, say P and @. Then the abscissa (the -coordinate) of the midpoint of the line seg~ ment PQ jis the abscissa of V. Since P and @ are the intersections of the line y =k with the parabola, the abscissas of P and Q must satisfy the equation attbrtomk or attbrte— k= Its roots are: fo Zt yvd md ve where D = BF — dae is the discriminant. tte bathe the abscissa of V = 2 = — 2, and the DY Nag doe erinaeat¥ o(—2) +0(-2) ten Sten tect if @ > 0, the ordinate of Vis the minimum value of the function ax + bre and if a <0, it is the maximum value, 42. If 2°” = 2, then the exponent, which is aguin 2" is also 2, and we have 2°42, Therefore 2 = V/2. SOLUTIONS: 1950 EXAMINATION 85 48, Rearrange the terms: 1 apraed = 7% 38 2 # 1 tht 7 6 44. Looking at y = logar or equivalently x = b», we see that y = 0 for 2 = 1. That is, the graph cuts the x-axis at 2 45. d = n(n — 8)/2 = 100-97/2 = 4860 46. In the new triangle AB = AC + BC, that is, C lies on the line AB. Consequently, the altitude from C is zero. Therefore, the area of the triangle is zero. 47. By similar triangles, (h— 2)/h = 22/8, O/(2k +b). 48. Let P be an arbitrary point in the equilateral triangle ABC with sides of length «, and denote the perpendicular segments by Po, Phy Pe ‘Then —— Area ABC = Area APB + Area BPC + Area CPA = (ep. + am + ap.) = dele + ps + pd). Also, Arca ABC = 4s-h, where is the length of the altitude of ABC. Therefore, b= p. + me + p. and this sum does not depend on the location of P. 49. Perhaps the easiest approach to this problem is through coordinate geom- etry. Let the triangle be A(0, 0), B(2, 0) and C(z, y). Then A1, the midpoint of BC, has the coordinates [(x + 2)/2, y/2l. ‘Therefore, using the distance formula, we have VETO A + C04 ‘This equation represents a circle with radius 8 inches and center 4 inches from B along BA; THE CONTEST PROBLEM BOOK tas extreme values we may have a— 6 = 2 and a , using the notation a, b, ¢ with its usual meaning. From the median formula, with c=2 and m= 3/2, we have +8— 9+, 2 —at =I. Using the given extreme values in succession, we have b= 5 and a=7, and b= 1 and a=1. Therefore, the maximum distance between the extreme positions of Cis 6 inches. These facts, properly collated, lead to the same result as that shown above, After the two-hour chase, at 1:45 P-M. the distance between the two ships is 4 miles. Let ¢ be the number of hours needed for the privateer to overtake the merchantman. Let D and D + 4 be the distances covered in t hours by the merchantman and the privateer, respectively. ‘Then Rate of merchantman-t = D, and Rate of privateer-t = D+ 4 3 hours, so that the ships meet at 5:20. 1951 Solutions Part 1 ‘The pedagogic value of this extremely simple problem can be enhanced by a discussion of necessary restrictions on the number-nature of M and N, the permissiblity of negative numbers, and so forth. . Let the di and 2w snsions of the field be w and 2v, Then 6 = z, w= 2/6, 2. A= wd = 2/18 Ande = Ho +p t. S = 1(13-10) + 2[2(13-5) + 2(10-5)] = 590. ‘Sy = 10,000 + 1000 = 11,000, 8; = 11,000 ~ 1100 = 9900, S)— S;= 1100. -(B) is the correet choice. If the dimensions are designated by 1, w, h, then the bottom, side, and frontareasare lw, h,Al. The product of these areas is Duh? = (lwh)* = the aquare of the volume. SOLUTIONS: 1951 EXAMINATION a 7. Relative error = error/measurement. Since 02/10 = .2/100, the relative errors are the same. 8 Let Mf be the marked price of the article. M — 1M = OM = new rice, To restore 9Bf to M requires the addition of 1M. 1M = #98), and 4, expressed as a percent, is 119%. 8 Let Ps be the perimeter of the kth triangle. Then Pizs = $Ps Sa P+ Pt Pet 1 1 = a04 5-304 : \e/ ei aX -o(i45ti+) % = 30 ~ 6a. 73) 10. (C) is incorrect since doubling the radius of a given circle quadruples the 11, The new series is Hah tarde; AS = at/(1— Fr). 12, In 15 minutes the hour hand moves through an angle of 4 of 30° = 74°. ‘Therefore, the angle between the hour hand and the minute hand is 22H. 18, In one day A can do 1/9 of the job. Since B is 50% more efficient than A, B can do (3/2)(1/9) = 1/6 of the job in one day. Therefore, B needs 6 days for the complete job. 14, (C) is incorrect since some terms (primitives) must necessarily remain undefined. 15, nt — n= (n— 1)(m)(n + 1), for integral values of 7, representa the product of three consecutive integers. Since in a pair of consecutive in- togers, there is a multiple of two, and in a triplet of consecutive integers, there is a multiple of three, then’ n? — 1 is divisible by 6. Part 2 16. The condition ¢ = ¥#/4a implies equal real roots of f(z) = 0 for real coefficients, i.e., the curve touches the z-axis at exactly one point and, therefore, the graph ia tangent to the -axia, 17, (A), (B), (©), and (E) are of theform y = kr or zy = k, but (D) isnot 18, 10. Ie Rae 5. After simplification, we have 2 P= RAVE, 1 = B16; THE CONTEST PROBLEM BOOK Let the factors be Az+ B and Cr+ D. Then (Az + B)(Cz + D) = ACz + (AD + BC)z + BD = 22 + ar + 21. AC = 21, BD = 21, Since 21 is odd, all its factors are odd. There fore each of the numbers A and C isodd. The same is true for B and D. Since the product of two odd numbers is odd, AD and BC are odd numbers, @ = AD + BC, the sum of two odd numbers, is even. ‘Such a number can be written as P-10' + P= PUI? + 1) = P(1001), where P is the block of three digits that repeats. -.(B) is the correct choice. ea yetey y= L(t) et ety te erway ~ sty (t+})- pb ttre b (©) in not always comet since, if + is negative, 22 < ye Since logu(at ~ 15a) ~ 2, then a ~ 162 = 10% = 100, ‘The solution set for this quadratic equation is. (20, —5} | Since Vi= ath, we must have x(r +z)! = w%(h +2). “z= (f¢ ~ Brh)/h. For r= 8 and h= 3, wehave z= 5b. art —a(2%) _2t-at— 2-9" 2-2-1) _7 Let 1 be the side of the square, a, its apothem; then sf = 4a; and since 2a, = 01, we have 4a! = 8a, or a, = 2 Let o be the side ff the equilateral triangle, its altitude and ay its apothem; then 83/4 = Ber. Since h = Bar, ands: = 2h/V/5 = bar/V/3, we hhave (360i/3)V/3/4 = 3-6ay//3, and a: = 2= a. = m(m +1) = 0. For equal roots the discriminant D = 1+ 4m(m+ 1) = 0 = (2m +1). m=} 7. We may eliminate (A) and (D) by proving that corresponding angles sre not equal. If (A) is false, (B) is certainly false, We may eliminate (C) by placing the point very close to one side of the triangle. Thus none of the four statements is true. k= 1/16; 36/9 = (1/16)-(1)-(V8); = V = 32 (mph). Let the ratio of altitude to base be r. ‘The triangles in the figure satiafy condition (D) but have different shapes Hence (D) does not deter- mine the shape of a triangle. 30, 3 34 36. SOLUTIONS: 1951 EXAMINATION 80 1 AL zat a! 42 = 16 (inches); leo 2 00-7 7 2 = 16 (inches). Let 7 be the number of people present. Single out a particular individual P. P shakes hands with (n ~ 1) persons, and this is true for each of the persons. Since each handshake is between two persons, the total number of handshakes is n(n — 1)/2 = 28. xm = 8 CC, 2) = nfm ~ 1/2 = 28. 2. The inseribed triangle with maximum perimeter is the isosceles right triangle; in this ease AC + BC = ABY2. Therefore, AC + BC < ABV. (2 = Br = 0) o9 (at = 22) oo = Bet 1 = 1) baer 0. ‘general, +:(C) is the correct choice In general, a” =, with suitable restrict let 10°” = N. Taking logarithms of both sides of the equation to the base 10, we have logy7-logiel0 = logueN. “N= 7. ” z= aly ns on a and Nj Since c= a’, c= a’ cay = @. Part 3 ‘To prove that a geometric figure is locus itis essential to (1) include all proper points, and (2) exclude all improper points. By this eriverion, (B) is insufficient to guarantee condition (1); i.e., (B) does not say that every point satisfying the conditions lies on the locus. 0 THE CONTEST PROBLEM BOOK 31. Let_N denote the number to be found; then we may write the given formation ss N 1005 + 9 u+8 =~ 2a, +1. Hence N+ 1 = 10 + 1) = (a4 + 1) = «+ = 2, +1), ie, N +1 has factors 2,8, 4,--+, 10 whose least common multiple is 2.3.5.7 = 2520. «N= 2510, 88, .03 = 600/z and 02 = 600/z; <2: = 20,000 and 2 = 30,000; 2 ~ 1, = 10,000 (feet). 89, The average rate of fall of the stone is 1 5 (init rate + fina rate) = 3 (0+ Since the distance d_traveled by the stone and the sound is the same, d= ry = rele. The times of travel are inversely proportional to the rates of fall: ri/ry = t/t, a 77 “Te 49. Since +2 (2+ 1-241) ond Pola @- De +e4H, the given expression, when simplified, is 1 (with suitable restrictions on the value of 2). st~Tlseoonds); +: 16 ~16-7" = 784( feet). 4, ce the y-ifferences are 2, 4,6,8, © quadratic function is euggested, thus eliminating (C). A quick check of the other choices shows that (B) in correct; ‘Make the check with each of the choices given. Mam Vide, P= 1+, 2-24-10, ond 2~ 162, sl and rem’, wehave Lyd ress g and rem Gy wehwwe atm —e . The area of the large square is 2a, where a is the radius of the circle. ‘The area of the small aquare is 4a#/5. The ratio is 2:6. From the given information it follows that for the right triangle ABC, (o/2 +O = 25 and oF + (6/2 = 40. sa = 36 and B= 16 EHH = 2 re = 2/78. 92 THE CONTEST PROBLEM BOOK 50. Let ts, tr, & be the number of hours, respectively, that the car travels forward, back to pick up Dick, then forward to the destination. Then we may write 25th — B5-te + 5:ts = 100 for car Bt Sle + 25-b = 100 for Dick Qt + Set St = 100 for Hany. ‘This system of simultaneous equations is equivalent to the system -bt oe 4 bth t b= 20 Bhat ht bam whose solution is t= 3, = 2, f= 3. Hence 1, + f+ f= 8 = total number of hours. 1952 Solutions Part 1 1, Since A = r7* and is irrational and r is rational, A is irrational. (The product of s rational number and an irrational number is irrational.) 20-804 30-70 _ 7, a (6-643) 4, For P— 1 pounds the charge is 3 cents per pound. For the first pound the charge is 10 cents. «.C = 10 + 3(P ~ 1); 2, Average ‘The charge for euch pound is 3 cents with an additional charge of 7 cents for the first pound. “.C = 3P +7 = 3P — 3 + 10 = 10-+.(P ~ 1). Sym metd, maBIb A BES 3, cy marth Subst tuting, -6 = 3.045; .b= —6; ay = 32 ~ 6. This equation i satisfied by (3,3) SOLUTIONS: 1952 EXAMINATION %3 6, The wots are (7 VAT FBG)/2_ and their dierece is THVS_1-VB_ yg yp T-VB_1+VB__ yg SO = VE or EE S = - VE. Of the given choices, (B) is correct. 1 ay O74 UA) “E+0 8, Two equal circles in a plane cannot have only one common tangent, CD CO 1 tty 0 me mb mca, mem Mn+ ca); bm sa/im + a0) wand, weg, wtuna(b+d) 10° 20° 10 * 30/7 0 4 AUTO) + OAH ~ 5 ~ "med: 11, The value x = 1 makes the denominator zero, Since division by zero is not permitted, (1) is undefined, so that (C) is the correct choice. 12, Let the frst two terms be @ and ar where 1 9%, the triangle is acute, so that (C) is a correct choice. A check of (D) by the Law of Sines or the Law of Cosines shows that it is incorrect. (B) is obviously incorrect by the Law of Sines, Note: The elimination of (B) and (D) involves trigonometry, and so, in part, this problem is beyond the indicated scope. Part 2 0 1 and h isreduced by 3 or 8”; we (Uioh see w= (04 2)(0— gat) = + n= lot — a ee £90 7 100 7 Toop 7 O And 1 = Orr (per cent). Let br the cont, I the at rc, th ing ie ad the marked pce, Ow 41, $= C418, 5 = $a 4 4y od : of fet = (C) is the correct choice log p + log g = log (pq). Thus we must have py = p+ @ =p = /(a~ 0) Since BD bisects angle ABE, ) we have 42 _ AB; ° DE ~ BE since BE bisects angle DBC, « we have DE _ BD 8 © Hence SOLUTIONS: 1952 EXAMINATION w.2—3y +:(E) is incorrect. a 21, Bach such angle is the vertex angle of an isosceles triangle whose base angles are each, in degrees, 180 — (n — 2)180/n = 360/n. svangle = 180 — (720/n) = (180n — 720)/n = 180(n — 4)/n. Bertha wsi; : t= VRB. iz 0 ; reuse m-1), 4m 21 The eqn eqs to a= (64 2E2)e4eBotoa ‘Since the coefficient of is equal to the sum of the roots, it must be zero. - 7 ma —a = We have b+ Mita 0. bm + d+ 0; ince AB = 2 and AC = 12, BC = 16. Since ABDE ~ ABCA, ‘Ares ABDE _ 10" eee ~ ig: But Area ABCA ~ 4-12-16 ~ 96 + Area ABDE = 374, fand the required ares is 96 — 37) = 58}. ‘Note: the method here shown is based on subtracting the area of ABDE from that of ABCA. It is a worthwhile exercise to find the area of the ‘quadrilateral by decomposing it into two right triangles. 25, Since the times are inversely proportional to the rates (Gee 1951, Problem 39), 1080/8 45 1000/8 5. ~ 08 and 27. 29, 30, al. THE CONTEST PROBLEM BOOK Let Py and P, be the perimeters of the smaller and larger triangles respeotively. We have = 83/2 and 9 = 2r/V3. WAN 2 Pi = 2G and Py = 3rVi; 2 PuPs (©) is shown to be the correct choice by direct check. Note: The fact that the y-differences are 4, 6, 8, and 10 ean be used to eliminate (A) and (B) immediately. Let KB=2 and CK = y. Then 8 —y) = 2(10~2) and FAG PER BOA ny = B5/ 10 = (15/2) _5 35, t $ and 10 8, 0 that (A) is the correct choice. Using $= 3 la + (m~ 1d, wehave P20 + 9d) = 4-320 + 40 daa sad = 12 Single out any one point A. Joined totheremaining 11 points, A yields 11 Tines. Since a line is determined by two points, we have for the 12 points, }-12-11 = 66 (lines); © (12,2) = 412-11 = 66, Time = distance/rate; <= 30/x. For a given perimeter the circle has the largest area; Jet P be the common perimeter, A: and Ay the areas ofthe circle and the square, respectively. Then pe P = 2er, Aap BP P= 4, Ani Since 4e < 16, Ai > SOLUTIONS: 1952 EXAMINATION a 34, Let the original price be P. ‘Then Py = ( +5 )P. = (a) = (+)? = 1 __ 10000 7/100} ~ 10,000 — an M24 VD + VE 2+ VE VIO, Wt+tvVa- V5 (V24+ V+ V5 28 : 2+ VE+VO Vo_2V5+6+2V5_ 3+ Vet VIB 276 Ve 26 Wanaars Part 3 #41 _ (e+ -241) -z+1 RAT + De) aT frat lim, 21 -3 tin 3 2 For continuity at z= —1, we must define #+i_ ia eT Note: This problem is beyond the indicated seope. 37. By symmetry, the required area is 4(7" + 8). T = 43 = 8V3, (30/360) 48! = Ste; B2V5 + ide. 38. 1400 = $-50(8a + 8B). a+b = 7. This indeterminate equation ‘satisfied by the following pairs of integral values: 1, 6; 2, 5; 3, 4: +: (D) is the correet choice. 39, P+ wt ad, 1+ w= p/2, (1+ wt = B+ 2lw + wt = p/s, lw = p/4 — a (L— wh = BP - ho + wt = 2d — /4, and toe Vera pn 8 THE CONTEST PROBLEM BOOK 40, We are told that the values of f(z) listed correspond to S02), fla + W), fle + 2A), + fle + 7h). given by See +) Se) = ale + WF + WE HM He- (+b +e) = Bakr + ab + bh Since this difference is a linear function of, it must change by the ‘same amount whenever 2 is increased by h, But the successive difer- ences of the listed values, 3844 3960 4096 42274356 4480 4624 4761 are 12517 SLD 183 SST 0 that, if only one value is incorrect, 4227 is that value, AL VEU = OF OMAR ATO, A +O RB 2318 ~ 127 — 36 = 0; =6 42, D = PQQQ +++ = 04 ++ abs -++ bby = by =++ - $0 (A), (B), (C) and (B) are all correct choices. To check that (D) is incorrect, we have 0D — 10D = PQ-P. «. 10(10' ~ 1)D = PQ ~ 1), 43, For each semi-circle the diameter is 2r/n, and the length of its are is ‘rr/n, The sum of m such arcs is n-xr/n = er = semi-circumference. Observe that the difference between successive valuet 44, Given 10u + ¢ = ku +0). Let 108 + w= m(u + 0), MC+u) = E+ mut); aktmall; am 45, The Arithmetic Mean is (a + b)/2, the Geometric Mean is -/ab, and the Harmonie Mean is 2ab/(a + }). The proper order for decreasing magnitude is (E); nak Since (a — b)F > 0, we have at + b> 2ab; =. at + Dab + OF > dab, a+ b> 2Vah, and (a + d)/2 > Vab. Since ot + 2ab + > 4ab, we have 1 > dab/(a + WY; + ab > 4a°B'/(a + 6), and v/ab > 2ab/(a +b). 46. Area (new) = (d+ Did —D = @— P= wh +. (C)is the correct choice. 47, From equation (1) = y*. From equation (2) = 264, 2— 2241; a2 z z + y+ = 16. This equation has one 9 From equation 3) #21 integral root y = 3. ay SOLUTIONS: 1953 EXAMINATION ~ 48. Let. f and + denote the speeds of the faster and slower cyclist, respec- ‘The distances (measured from the starting point of the faster eyelist) at which they meet are Q) f-r=k-+ er when they travel in the same direetion, and @) J-t= k= et when the slower one comes to meet the other. From (2), f-+ a= k/ts from (1), f—e= kir. ayek gh a MetO, gab b Mead, forte and . 49. By subtracting from AABC the sum of ACBF, ABAE, and ACD and restoring ACDNy + ABFN; + AAENs, we have ANNs. ACBF = ABAB = AACD = 4AABC. From the assertion made in the statement of the problem, it follows that ACDN, = ABFNs = AAENs = +-4AABC = dA ABC. + QNNaNs = DABC = 3-4AABC + 3-grQABC = $A ABC. 50. Rearrange the terms: Ae ee settle ty a = ME ME VB 5S =S += KG + V2). 1953 Solutions Part 1 1, The profit on each orange sold is 4 — 3} = Jeent. «. $n = 100, n = 150. 2. D= Dy + Di— DiDs, D = 415-3 = 32. |S = 250 — (.32)(250) = (.68)(250), that is, 68% of 250. (See 1950, Problem 22.) 3. 28+ yf has no linear factors in the real feld.2* + yf = (x + iy)(z — iv) in the complex feld. ‘Note: This problem is probably beyond the indicated scope. 100 THE CONTEST PROBLEM BOOK 4, Set each factor equal to zero, and solve the three resulting equations, ‘The roots are 0, —4,—4, and 4. (The factor 2° + 8r + 16 = (x + 4)* hhas the double root "~4.) 5. 2 = 6 = 6.6" = 366. 6. 31(60 + 1) ~ (a + 5)] = 3 (4 - 4) = 1069 ~ 1), 7, Multiply numerator and denominator by VaF a and simplify, (D) is the correct choice. 8, For intersection 8/(u? + 4) cia Be bbe = ate — Be bw) r0. ober t, aeee 10. The circumference of the wheel is 6x. © N = 5280/6r = 880/r. 1. y= 2an, Ce = Bath + 10), 2 Ch Ch = 20K ~ 60 (ee. 12. Ay/As = wH/x6 = 4/9, 13. A = 4bh = $A(bi + ba); n+ br =b = 18. 1m = Hb, + by) = 9. 1s. tach assertion is realized in the accompanying figure. Hence, none of these statements is false. (8) a 15, Designate aside ofthe original square by «Ten Ge radius ofthe inmenbed circle a «/3. A sige of the second equsre Gnsribed in the circle) is r~/3 = 4/-/3. ‘Therefor, the aren of the second square is £72, while that of the orginal square is a. (B) i the correct Chole. SOLUTIONS: 1953 EXAMINATION 101 Part 2 16. S = Selling price = cost + profit + expense = C + .10S + .158; 2 $= 4/3C; + (D)is the correct choice. rlyyt wnat Lat be the amount invested at 4%. Then .O4y = .06(4500 ~ y) andy = 2700. Therefore the total interest is .04(2700) + .06(1800) = 216, and (216/4500)-100 = 4.8%. Wattdn tart 4) 40 = (+ 2) — Bet = (2 + Be + 224 — 2e + 2). 3. (3) 2a a 10 (a) = Ba ante dei meee wtrerieel(eade rj] By -2+y-4) = 2 +y- 6) 21 # Br +6 = 10 = 10 or Br 4 = 0; 22. 27-75-95 = 3-3". = 3", logs 3" = 4b 23, Multiply both sides of the equation by Vz 10. 2+ 10-6 = 5V¥z+10, 2 — Iz — 234 = 0, x = 26 or -9 9 fails to check, ic, itis an extrancous root. 24. 1000 + 100(6 + ¢) + be = 1000! + 1000 + be. For equality we must have b+ = 10. 4or-1 Bar = art art, tre ar=(vE-12 26, Lot be the required length. Then 23/15 = 2/3 eta 10; 2 BYE 2. rtTt hte, Sa elt 0/0] = 49/3 28, The bisector of an angle of the triangle divides the opposite sides into segments proportional to the other two sides, ie, y/e = x/b. It follows that 2/b = @ + p)/lb + 0) = a/(b + 0). 102 THE CONTEST PROBLEM BOOK 29. (D) is the correct choice. Consult seetions on approximate numbers for the explanation. Note: For a full understanding of the topio, a knowledge of ealoulus is needed. 30. B pays A 9000 ~ 75-9000 = $8100. ‘A pays B 8100 + 24-8100 = $8910. ‘Thus A has lost 8910 — 8100 = $810. 31. 80 ft. per second corresponds to 1 click per second. In miles per hour, '30-60%/5280 = 225/11 mph corresponds to 1 eliek per second. 1 mph corresponds to 11/225 clicks per second, which is approximately 1 liek in 20 seconds. So z miles per hour corresponds approximately to clicks in 20 seconds. 32, The diagonals of the quadrilateral formed lie along the two perpendicular lines joining the midpoints of the opposite sides of the rectangle. These diagonals are of different lengths, and they are perpendicular biseetors of ‘each other. Therefore, the figure is a rhombus; the sides of the quadrilateral can be proved equal by congruent triangles. Also it can be proved that no interior angle is aright angle. 33. Let x be one of the equal sides of the triangle. 2p = 22 + 2/3; 2» Lt 4 O-4V8 neg - THAR Gaya OPN? M, aa 35. Sz + 2) = @ + 2z +2 —- 1)/2 and Se+ l= @+D@4+1-1/2 So @ + D/2= fet Wiz; fla + 2) = + Wat Vie. ‘Note: The symbolism here is beyond the indieated scope. SOLUTIONS: 1953 EXAMINATION 103, Part 3 36. By actusl division, the remainder is found to be 18 + m. Since x — 3 ‘factor, 18 +m = 0, and, therefore, m= —18. s. (C) is the cor- eet choice; by the Factor Theorem Converse, 36 ~ 18 + m= 0. «m= ~18" BT. Let x be the distance from the center of the cirele to the base and let Then h= r+ be the altitude of AABC. MEO = 1 he Vi36, fests =F + mF BM Dhr + (9 + m/2h + 18) 2W/ TS = MAV185/270 = 8VI5/5. 38. F13,f(4)] means the value of +a when a=3 and b= 2, namely, 7. Note: This problem is beyond the indicated seope. 39. For permissible values of a and b, logs -logy a = 1; Tet 1 = log, and y = log a. Savebeg aye; a= and =a. 2 loge blogs a = 1. 40. To contradict the given statement, preface it with “It is not true that +++", Therefore, the negation is (C); in symbols, given Falz M |x is honest). ~ Vs = Gale eM | is dishonest). 41, OB = 400 rode. 2 = 300 + (400 — 2}, 8002 = 250,000. cB = B12} (rods); 300 (B) is the correct choice. os 104 THE CONTEST PROBLEM BOOK 2.6m 4 OF = 4 43. Lets be the price of the article, m the number of sales. Then (+ pall ~ dn] = am, p~ d ~ pd = 0, d = pf +p. 4, Let the true equation be 2 + br + ¢ = 0, the equation obtained by the first student be 2*+ br +c’ = 0, and the equation obtained by the second student be 2° + b'r +c = 0. etree PHUrte afthete 45, For a +b, Arithmetic Mean > Geometric Mean. For a = b, Arith- ‘metic Mean = Geometric Mean. (See 1952, Problem 45.) = (a + by/2 > Vab- 4. VETER = stl— fees. 47, Forall2 > 0, 14+2< 10. 2 log +2) 7— 2 Adding x + 3 to both si 3 10 have 3¢ > 10 or 2 > 2. s of the inequality, we 7 Similarly, XB’ < 90°, 4.0’ < 00° 108, THE CONTEST PROBLEM BOOK 20. By Descartes’ Rule of Signs (B) is shown to be correct; solve to find the roots —1, —2, — graph y = 2+ et + 11 + 6. 21, ava + Jz = 6, Squaring, we obtain 42 +8 +4 = 25. Multily both ie of the equation by and obtain dat ~ fe + 4 =0 22, Adding, we have Bt ~ 2-4-2 %e-2He41) G@FNE- ~ CFVE-D for values of © other than 2 or —1. 2 s Lo, 2. s=c+leo; ati w+ a ath 24, Bet — afk — 1) +8 = 0. For real, equal roots, the diseriminant 1-0 and E148 R= 9 or -7, 25, The product of the roots is (a — b)/a(b — ). Sinee one root is 1, the other is the fraction shown. 26, Let x = BD and let r be the radius of the small cirele. Draw the line from the center of each of the circles to the point of contact of the tangent and the cirele. By similar triangles, etre stor r o mpm 6k and nt = 7h = kom uk int Tor ~ BEE” Hak Ha SOLUTIONS: 1954 EXAMINATION 109 29. In any right triangle (see ‘secompanying figure), we have ee ee ally Since $= 5, it follows that 80, Denote by a, b, ¢ the respective number of days it would take A, B, C to complete the job if he were working alone; then 1/a, 1/6, 1/e denote the fractions of the job done by each in one day. Since it takes A and B together two days to do the whole job, they do half the job in one a2 3 Bl. $BOC = 180 ~ 1(4.B + 40). But (4B + 40) = 4-140 = 70, %BOC = 110 (degrees). 82. ot + 64 = (+ 160" + 64) — 16% = (4 BF ~ (Az) = (et + de t+ OY — Ae +8), (Gee also 1953, Problem 18.) 83, The exact interest formula is complex. Approximately, he has the use of $50 for 1 year. 6-100/59 = 104. -. (D) is the correct choice. 34, 1/8 is the infinite repeating decimal 0.333 --- . Thus 1/3 is greater than (0.33838833 by the amount 0.00000000333 1/3-10%; ogamanaa = 9:10 4 310 4 oo ¢ ae10% = SOS 10) Ta a(1- 10%) _1_ 10" settee cee 3 no THE CONTEST PROBLEM hOOK c+ YEP RPAH Atd, VPP OFS HAtd— PHRF Det OL E+ Bhd — Ihe — Bde, Dee + de = hd; sem hd/(h + d), 40 2 : 9, B= SG =F. (see also 1951, Problem 39.) BT. Am= Ap+ Xd, Xd = Xq— Am (there are two vertical angles each Am) m= Apt Aq ~ Ams Rm = Map + Ho). 38.363 = 195; B= 55 “+ = log 3 = log 5 = log (10/2) = log 10 — log 2 1 = 03010)/0.8771 ~ 1.47. 30, Let PA be any line segment through P such that A lies on the given circle with center O and radius r. Let A’ be the midpoint of Pa and let 0” be the midpoint of PO. To see that the required locus is a circle with center 0’ and radius 4r, consider the similar triangles POA and PO‘A', For any point A on the given circle, O'A’ = $04 = dr. :-(B) is the correct choice. watst=(a43) ~a(04!) = avi-svi-o, 41. 8 = ~ (8/4) find one root, —3, by trial. The reduced equation is 4z* — 202 — where the sum of the roota is 5. $=5—-3=2, 42. From the graphs, or from a consideration of the positions of the lowest Points, it is found that (D) is the correct answer. (See 1950, Problem 41.) SOLUTIONS: 1954 EXAMINATION ut 4.¢ =a —1-4+b—F (see 1950, Problem 35). = 10 = a +b ~ 2; Pratbte=I0+2410= 2 44, We seek an integer whose square is between 1800 and 1850. There is one, and only one, such integer, namely 1849 = 43. <. 1849 ~ 43 = 1806, the year of birth: let_y be an integer between Oand 80. +. 1800-4 y + 2 = 2%. For 2 tobe integral, the discriminant 1 + 7200 + 4y must be an odd square. ‘The value y = 6 makes the discriminant 7225 = 85%. The year of birth is, therefore, 1800 + 6 = 1806. 45, Let d denote the distance from A measured along AC and let U(d) denote the length of the segment parallel to BD and d units from A. ‘Then by similar triangles, we have: (1 _ (2) BD a” G/ac’ Xe s, For d> GPL _ 2) BB xe fet ore aoe oa ae 2k(AC ~ d) = —2kd + 24AC. ‘The graph of I a3 a function of d evidently is linear ind. Its slope 2k is positive for d < AC/2; its slope —2k is negative for d > AC/2. Hence (D) is the correct choice. 46. The distance from the center of the circle to the intersection point of the tangents is 3/8. 3 Dat a1. Since HP = VPS, ap = au + WT = P+ VP 8 ang TB = 4B - AT therefore, 2 — pz + g = 0. P= YEH rhe mired eqation ua THE CONTEST PROBLEM BOOK 48. Let x be the distance from the point of the accident to the end of the trip and R the former rate of the train. Then the normal time for the trip, in hours, is 2 +R anim: Considering the time for each trip, we have Ly stk, gt bee Ray eee ee | and %,1,2-9 24R emt Gaye cmese. 540 and R= 00; = the trip is 540 + 60 = 600 (miles). 49, (2a+ 1) = (26 + 18 = dot + da $1 — 4b — ab 1 = Mala + 1) - 06 + 1) Since the product of two consecutive integers is divisible by 2, the last expression is divisible by 8. 80. 210+ x — 12r = 84; Le = 125; x = 126/11, 126-00/11-30 ~ 23 and 210 + 2 + 84 = 122; Lr = 204; "2. x = 204/11, 204-60/11-30 ~ 53. the times are 7:28 and 7:53, 1955 Solutions Part = 0.000000375. _«. (D) is the correct choice. Cee a 1. $= 375; + B10 fit oer tt aerecerr ater 2, 150° — 3 -a0e = 1s74° = 137° 30 8. Lot A be the arithmetic mean of the original numbers. Then their sum is 104 and A(new) = (10A + 200)/10= 4 +20. (@B) correct choi 4. Multiplying both sides of the equation by (2 — 1)(¢ — 2), we have M-2az-2% n2H0, By = b/s, 16 = k/l, k= 16; ey = 18/2, y = 16/8-8 = 1/4; wr = 2i/zt, Wn = 8/1; = 1/4. SOLUTIONS: 1955 EXAMINATION us 6. Let _n be the number bought at each price, and 2 be the selling price ofeach. Qnz = (10n/3) + 4m; z= 11/8, that is, 3 for 11 cent, 7. Wa = Wi — Wi/5 = AW/S. +. Wi = 5Wa/4. ‘The increase needed ig Wy/4 or 25% of Ws 8. 2 ~ Ay = (e+ 2y)(e — 2) = 0; Each of these equations represents 9. This is a right triangle. For any right triangle it ean be shown (see 1950, Problem 35) that st 2y = 0 and 2 —2y = 0, straight line. anrtb— steadtb-ca8+15-17=6 wd re 10, Tota is moving fr hour andi tet for nim minutes ot Th hours, f+ 11, The negation is: It is false that no slow learners tend this school. There- fore, some slow leamers attend this school. 12, V&E=T = 2- VER, and b2-1=4-4V7=TH 2-1 fed AVE OT, 2 1 VF = 1. Squaring again, we eet BoWwtlazol, #4 2=0, r= 1 or 2 Since 2 does not satisfy the original equation, we have one root, = (+ Vat oF ) 13. ot 14, The dimensions of Rare 1.le and 0.98; its area is 0.00e R/S = Sat/st = 99/100, 15, Let_z be the radius of the larger circle. Then s7!/rzt = 1/3, 2 = r/3, VB — 1 = (V5 ~ 1) ~ O78. Part 2 16, When a= 4 and b= —4, a +5 = 0. Therefore, the expression becomes meaningless for these values. 17. log z — 5 log 3 = log (2/3"); = 10* = 2/38; 18, For real coefficients a zero discriminant implies that the roots are real and equal. 2.48, ua THE CONTEST PROB 19. The numbers are 7 and ~1; the required equation is, therefore, #-6r+7=0 20. The expression V25=F + 5 can never equal zero, since itis the sum of 5 and & non-negative number. (By ¥/25 = @ we mean the positive square root.) (A) is the correct choice. ince phe = A, h = 2A/e. 22. Since a single discount D, equal to three successive discounts Di, Ds, and Dy, is D = Dy + D+ Dy ~ DiDs ~ DaDy ~ DsD, + D,D.D (Gee also 1950, Problem 22), then the choices are: 20 + 20-4 10-04 02-02 +00 = 424 and 40 + 05 + 05 — 02 — 02 — 0025 + 001 = 4588. ‘The saving is .0345-10,000 = 345 (dollar). 28. The counted amount in cents is 25 + 10d + 5n + ¢ The correct, amount is 25(q — 2) + 10(d + 2) + 5(n + 2) + (c ~ 2). The dit ference is —252 + 10 + 5e— z= —11z,~. Mz cents should be subtracted 24. The graph of y = 424 — 12 ~ 1 is a vertical parabola opening upward with the turning point (a minimum) at (3/2, —10). (See 1950, Problem 41); algebraically, the turing point occurs at 2m 0/20 = (= 12/8) = 3/2, For this value of 2, y = 42" — 122 ~ 1 = ~10. 25. A+ It + Om (e+ Get +9) — (Ae) (at + 3)! — (Qa) = (28 = Bet Blot + Be +9), + (B) is the correct choice. 8 See 1951, Problem 5, Wt = (ra Bem pr 28, For 2 0, axt+ br} om ast ~ br +e; for 2=0,ar ¢ bre = ar —bete. ". There is one intersection point (0, ©); -. (E) is the correct choice. SOLUTIONS: 1955 EXAMINATION us 29, First, draw the line connecting P and R and denote its other inter- sections with the circles by Mand N; see accompanying figure. The ares MR and NR contain the came number of degrees; so we may denote each are by z. To verify this, note that we have two isosceles triangles with a base angle of one equal to a base angle of the other. “ANOR = 4 MO'R. ‘and the sum of angles 4 PR and BPR is SEPA =e 40. ‘The desired angle is 360° — BPA = 360° — (¢ +d) = (180° ~ ¢) + (180° ~ a) sath, 30, The real roots are, respectively: 43; 0 and 2/3; and 3. =. (B) is the correct choice, BI. Let Ay and As denote the areas of the small and the original triangle, respectively. The median m of the trapezoid is the arithmetic mean of the parallel sides, that is, m = 4(b + 2), where & denotes the shorter parallel side of the trapezoid. To find b, observe that cage esata 1 Git prys sb VE and ma 3 (v2 +2). 22. D = 48 — dae a Baan = ae, a/b = W/e 116 THE CONTEST PROBLEM BOOK 33. Let x be the number of degrees the hour hand moves in the time in- terval between 8 a.m. and the beginning of the trip, and let y be the number of degrees it moves between 2 p.m. and the end of the trip. The ‘number of degrees traversed by the minute hand during these intervals is 240 + x and 60 + y + 180, respectively. Since the minute hand traverses 12 times as many degrees as the hour hand in any given interval, we have tien mots, 2-80 and ay = e044 160, y= 2, hat ithe uh st many depp th 8 a he egang bp Shad a lee Sl ten ee Spun al al psn ef he hd Sie ro 1, har ane emt che 34. The shortest length consists of the two extemal tangents 1 and the two circular ares ly and fy = 2, b= MO 26.9 = 126 = 120», t= 6V3, = 123, b= HP Oe: % vo: length = 124/3 + Mx 38. Th int boy taker 2+ 1 marie, laving #1 marble, The scond 1 vay ( 1) ein eit ves many ey 3 have 2(5 1), 9 dat ws indeterminate integer of the form 2+ 6a, with a= 0, 1,2, fe. n may be any even Part 3 36. The area of the rectangular surface 10-2r = 40. 22 and y= V5; = the depth is 3 — V7, gees or 3+ V5. LLP 37. The original number is 100k + 10¢ + w. The number with digite re- versed is 100u + 10¢ + h. ince A > u, to subtract we must add 10 to w, ete.) = 100(h—=1) + 100+ 9) + u + 10 1000 +10 +h 100k 1a) +90 + 10+ uh Since 10+ u—h=4, h—1—u=6 and therefore, the digits from right to left are 9 and 5. 40, 4 2. SOLUTIONS: 1955 EXAMINATION nT Solving the system: Hat b+e) +d = 29 Votetd) +a=B Hetd+a)+b=21 Ad +a4b)+e=17, we obtain a = 12, b= 9, ¢= 3, d = 21. Thus (B) is the correct choice. ‘The least value occurs when x = —p/2. For x = —p/2, v= (-P/A)+q= 0; + q = B/A Bee 1950, Problem 41.) A non-unit fraction b/d is changed in value by the addition of the same non-zero quantity x to the numerator and denominator. -. (A) is the correct choice. a-R GAR (Gee also 1054, Problem 48.) ut ford —a4/? then atta? ace We 1; ne = bla ~ V/a. 9 ,1,¢-R-% 41+ D454 Sajid as ae ithe ye @ + and ett w+ Y= e+ D5 and =: (e+ 1) = 1. This last equation has one real and two imaginary roots. 1 (Eis the correct choice. KOBA = 2%; XOAB = 2y; and «2 = 4OAB + y = By, Let the two series be a, ar, art, -++ and 0, d, 2¢, sa=landr+d= 1, e+ 2d =2 att =) _1@"—1) * zo S 1023, S = FO + (n— al = Bo + (=D) = -45. 023 — 45 = 978. We are required to solve the system of equations: De + 3 Solve the first two simultaneously, obtaining 2 = 2, y = 2/3, Since these results are consistent with the last two equations, the solution is (2, 2/8) and (B) is the correct choice. 118 THE CONTEST PROBLEM BOOK 47, For the equality a + be = a? + ab + ac + be to hold, we must have anad+ab+acorl=atb+e, 48. (A) jis true because FH is parallel and equal to AE. (C) is true be- cause when HE, which is parallel to CA, is extended, it meets AB in D; DC and BH are corresponding sides of congruent triangles ACD ‘and HDB. (D) is true because FG = FE + EG = AD + 4DB = 148. (B) is true beeause G is the midpoint of HB. (FE is parallel to AB and E is the midpoint of CB.) (B) cannot be proved from the given information. Challenge: What additional information is needed to prove wy? 49, Since _y = (a — A)/(e = 2) = (e = 2)(e + 2/@ - 2) 2 + 2 (for = #2, ie y #4), then y = (2 ~ 4)/(2 ~ 2) is. straight line with the point (2, 4) deleted. The straight line y = 2r crosses the first line in the deleted point, (©) is the eorreet choice. 50. Let 1 be the increase in the rate of A and d be the distance (in miles) A travels to pass B safely. The times traveled by A, B and C are equal d__ d= (30/5280) ~ (30/5280) _ (210/5280) wrt Oa 0 cy Solve the second equation ford. = d = 110/280 miles Solve the first equation for r. mph. 1956 Solutions Part 1 12422) = 2485100. 2S = C+ HC, and S = Cy Hr. 2 Ch = HS, = 1.00 and Cr 4S = 150. Si +S = 240 and C+C,= 250; = (D) is the correct choice. 3. 5,870,000,000,000 = §87-10", (587-10")100 = 587-10". 1 1 2 4. 35° 1000 + 3 - 3500 + 55 - 2500 = 500, 252 = 160, 2 = 64. 5. Around a given circle ean be placed exactly six circles each equal to the given circle, tangent to it, and tangent to two others. The are between two successive points of contact of the given circle and the outside circles is one-sixth of the circumference. SOLUTIONS: 1956 EXAMINATION ug Let a be the number of cows and b the number of chickens. Then 40+ 2b is the total number of legs and 4a+2= 14+ a+b), 2a= 14, a =7 (cows). Note: ‘The number of chickens is indeterminate. For reciprocal roots, the product of the roots c/a = 1 B= 8 B= 1 8TH O and z Note: 2" = 5°, but, otherwise, 2* = 5°. 1% ADB = 4% ACB = 30° LO + VOU ~ v8) 10 = VFN VI Vg a+ va) = V3) ¢ L-z y ~ & is the excess of y over 2, The basis of comparison is the ratio of the excess toy, namely, (y — 2)/y. Therefore, the per cent required is 100(y — 2)/v. PB-PC = PH. PB(PB +2) = 300; «. PB = 10. . Multiply both sides of the equation by 2* — 4 = (x + 2)(z — 2). 15 — et 2k 4; a+ 22-15 = 0; and =5 or 2 = 3. Each of these roots satisfies the original equation. 16. 17. 18. 19. Let 10m, 15m, 24m be the three numbers. Then 10m + 15m + 24m = 98, m=2 and 1m = 30. A(Qe — 3) + Bi2+2) fe — 11 ~~ @F DG - 3) Beta A(Qe — 8) + Ble +2) = 2 — 11. ‘Equating the coefficients of like powers of x, we obtain 24 + B= 5, -3A42R= 0; A Ben 10% = 5%, 10 = 5, 10-7 = 1/10" = 1/5, Let the height in each case be 1. 1— = 20-4; ak 19 THE CONTEST PROBLEM BOOK (log 2 — log 10) = log 2. 3010-1 21. Each line has 1 or 2 intersection points. Therefore, for both lines, there may be 2, 3, or 4 intersection points. 22. t= 50/r, ty = 300/Sr = 100/r = 21; +. (B) ia the correct choice. 23, Each of the roots is -/3/a. Since a is real, (C) is necessarily the correct choice. A. Let DAE = a. Then XBCA = 4180 ~ 30 ~ a) = 75 — (0/2) and 4DEA = 3(180 = a) = 90 — (0/2) = 4D. a+ $ADE + 2+ BCA = 180; 15 (degrees) Ba SFSH TH + Ont I= FGF Mt DY = mn +2). 26, Combination (A) determines the shape of the triangle but not its sie. All the other combinations determine both the size and the shape of the triangle. ‘27, The two triangles are similar and hence A\new)/A(old) = +. (C) is the correct choice. 28, Let 42 be the daughter's share, 32 the son's share. Then z+ 3x = festate and 6+ 500 =} estate. — . Te = Gx + 500; sr = 800; and 18x + 500 = 7000 (dollars). 29, 28 + yf = 25 isa circle with center at the origin. xy = 12 isa hyperbola, symmetric with respect to the line x= y. points of intersection ‘are symmetric with respect to z = y. There are either no intersections, ‘two intersections (if the hyperbola is tangent to the circle) or four inter- sections. Simultaneous solution of the equations reveals thet there are four points of intersection, and that they determine a rectangle whet yi; nen, an tvS_W VE_ 63 aig (2a /et = 4, BL. 20 = 1-4 Ltt + 0% + the required number is 110,; 20 + 4 yields 5 and remainder 0; 5 + 4 yields 1 and remainder 1; ++ 20 becomes 110, SOLUTIONS: 1956 EXAMINATION 12 32, After 1f minutes each was in the eenter of the pool. After 3 minutes they ‘were at opposite ends of the pool. After 4} minutes each was the center of the pool. 33, It ean be shownt that 4/2 is not a rational number. It ean also be shownt that repeating non-terminating decimals represent rational num- bers, and terminating decimals clearly represent rational numbers with a ppower of 10 in the denominator. Hence a non-terminating, non-repesting, decimal is the only possibility for representing +/3. 34. By considering the special case n = 2 we immediately rule out choices 36. (B), (C), (D), and (B). We now show that (A) holds oye yeaa bee a leak where k isa product of three consecutive integers, hence always divisible by 3. If n iseven, k is divisible also by 2 (since n is a factor of 3), hence by 6, and nk by 12; if m is odd, is divisible also by 4 (since the even numbers n— 1 and m+ 1 are factors of ), hence by 12, A= 16-8 V5 = 128 V3. Lae S = K(K + 1)/2 = N* The possible values for N* are 1,2, 8, --, 9. For K to be integral, the discriminant 1+ 8N* of the equation K* + K — 2N¥ = 0 must be a perfect square. This fact redueee the possible values for N* to If, 6%, and 35%, Hence the values of K are 1,8, and 49. Note: There are ways of shortening the number of trials for N* still further, but these involve # knowledge of number-theoretic theorems. ‘The shortest way to do this problem is by testing the choices given, ‘The diagonal divides the rhombus into two equilateral triangles A (ehombus) = 2-4 VB 2, BAS"VS. Since the map sale is 400 rmiles to 1 inches, 1 inch represents $400 miles and 1 square inch represents (4.400)* square miles. The area of the estate is thus 23 | soot BENS. SOP = 1250 4/8 (equate miles) 1 See, ¢4., Ivan Niven, Numbers: Rational and Irrational lao in this series. 12 THE CONTEST PROBLEM BOOK 38, The segments of the hypotenuse are and % (Gee 1954, Problem 28). Using similar triangles, Z = Y/6, 7. #/¢ ee ae e read Bata e—(- I= 2 -1acte-lacta oa = (et IP—a@= 2+ 1=atltancta; = (+aye—a) eta since e-a= 1, $0.28 = gf + 2Vel; Vimo + Vet; 2 28 — Vem Vols and 28, +0. AL, Since y= 22, y+ y +4 = 1a + 22 + 4. Wet Oda pA $4; Bede; n=O. 42. Vita = y=3-1 r+4a2-3-2VF=341; 2.3 = —s/z— 3. Thisisimpossible since the left side is positive while the right side is negative. 43, Let the langest side be c. Then a+ b-+e < 12 But car and ar>a war >a 45, y= Lilo, ya —tmile _y, _y, mile, _1/2 BD” 2" 7) ru 49, SOLUTIONS: 1956 EXAMINATION 133 aad ™ D-172~ By the percent increase is 100 - 35 ~ 2% Solving for z wehave x = 1/(2N + 1). Since N is positive, 1/(Q2N + 1) is positive and less than 1, (A) is the correct choice. In day, machines can do } of the job and z+ machines } of the job. machines ean do} — 4 = § of the job in I day, and machine can do. #s of the job in I day. Hence, 18 days are needed for 1 machine to complete te job; (+ B)/z = 3/2; +. 2 = 6, that is, 6 machines ean do 1/3 of the job in 1 day, ete Let (3p + 25)/(2p — 5) = m, a positive integer. Then, 3p + 25 = kn, 2p— Sah 2 kn — 3) = 65 = 1-65 = 5-13, = k= 1, 65, 5, or 13 and 2n~ 3 = 65, 1, 13, oF 5 correspondingly 2pm 5+, 5465, 545, of 541 P= 3, 35, 5, of 9. choice (B) is correct. 4P = 40°; .£PAB + PBA = 180° — 40° = 140°, XTAS = 180° — 4PAB; XRBS = 180° ~ XPBA; ATAS + 4 RBS = 360°'— 140" 8 a Lt Since 04 en OB et 9 freee TAS ad Ras wept, §— ZX OAS + 4.0B8 = 3(220°) = 110°. Tan :AOB = 180° — 110° = 70°. ‘The number of degres in %AOB is independent of the postion of tangent ASB. 180° ~ 24. Since CA = CD, Aas) 0 4}[l80° — (x. ACB + 90°) = 31180" — (180° — 24 + 90°) Ana =A-X. x= 45° 18 10, nL 12. 13, “4 15, THE CONTEST PROBLEM BOOK 1957 Solutions Part 1 ‘The altitudes, the medians, and the angle-bisectors are distinet for the legs but coincident for the base. Hence, 7 lines, dQ = 4 b= 8, 2/2 k= 8. 1 = (ala +0) = 1-140 a, provided a 1, In this case a = 32 +2, b= x, ¢ = — sab + ac = (8x + 2)x + (Bx + 2)(—5). log a — log b + log b ~ log e + loge ~ log d — loga — log y + log d + log x = log (2/y). ‘Thedimensionsare x, 10— 2x, 42. V = 140r — 4828 + 42% FFor the inscribed circle r= h/3 = 9/3/6_p = 38 = 6VBr = (6-V/3)(4-/3) = 72. (ari r= VB = 4V3) . The numbers are easily found to be 20, 30 and 50. -. 30 = 20a ~ 10; a= 2 2- (-2=2-16=-4. For a low or high point 2 = —4/4 = —1 (see 1950, Problem 41). When z= —1, y= 1. The point (=1, 1) is low point since the coefficient of 2 is positive 90 — (00° + 3-30)° = 224°; “(Bis correct. (See also 1951, Problem 12.) 10 = 10-10; 10 = 2.10- = 10-"(10 — 2) = 8.10-%, (A) and (B) are irrational. Of the remaining choices only (C) is between Vi and V3. Sine VF = FT (D) is correct roll and VEF EFT = [2+ 1), ‘The formula satistying the table is ¢ = 100, When ¢ = 2.5, # = 625. SOLUTIONS: 1957 EXAMINATION 125 Part 2 16. Since the number of goldfish n is a positive integer from 1 to 12, the graph is a finite set of distinct points 17. The longest path covers 8 edges, and 8-3 = 24 (inches). 18. ABM is a right triangle. By similar triangles, AP/AB = 40/AM. 2 AP-AM = A0-AB. 19, 1001, = 1-264 0:28 0.24 1-24 120= 164241 = 19, (Gee also 1956, Problem 31.) Tutt), goat 2.5 3(f5+ip)) A = 44%. ee 1950, Problem 27>) 21, (1) is the inverse, (2) is the converse, (3) is the contrapositive, and (4) js an alternative’ way of stating the theorem. the combination (3), (A) is correct. w. Visit ne-142VvE=Ttlesth Nave=t cde — "4 = 1; and vs de = 5. (This value of 2 satiaes the orginal equation.) 23. The intersection points are (0, 10) and (1,9). d= V(=IF FT = V2 24, (106 + uw)? — (10u + 0" = 99(0 — uF). This is divisible by 9, 11, f+ut—u + (B)is the correct choice. 25. From 2Ri(e,d), draw line segment RA perpendicular to the z-axis, Let O denote the origin (0, 0). % ¥ Riel bober ‘Sar He>b, ‘area APQR = area trapezoid OPRA — area AQAR — area OPQ tela + d) — dle — b) ~ Jab = Hac + bd ~ ab) We 0 implies x — 1 > 0. eB> hy | > 1, that is, 2>Lorz <1. Combiningthese, wehave = 0, 2< -1, 221, 30. The formula is $ = n(n + 1)(@n + 10/6; first let S = 1 and then let $= 1 + 2 1a +6248) , 8 and Peg 240049 Solving for ¢ and k, wehave ¢ = k= 1. 31. Let 2 be the length of one of the legs of the isosceles triangle. Then a ‘de of the square is given by 1 — 2r and this side must equal the hy- potenuse of the triangle (to form a regular octagon) eavial—de ae= @- VI2 32, Since nt — n= (n= nln + Nia + 1) = Lot + 1), where & is ‘a product of 3 consecutive integers, is always divisible by 6 at least (see 1951, Problem 15). We show next that in addition, »* — » is always divisible by 5. If we divide m by 5, we get a remainder whose value is either 0, 1, 2,3, oF 43 ie, nm Sotr, 7 =0,1,2,8 or & By the binomial theorem, (oq +7) = (5g +) (5q)* + 5(6q)% + 10(54)*2 + 10(5)** + 5{5a)r* + rt = 59-1, Sere where 1 is an integer. SOLUTIONS: 1957 EXAMINATION wr i divisible by 5. i divisible by 5. Ir=1, 5(V + 48) BV + 204) is divisible by 5. ‘Thus, forall integers n,n — n indivisible by 5 and by 6 hence by 30; (nt — 1) is divisible by 5 since n= 1 (mod p) where p is a prime. — n is divisible by 30. 38 OM Fm 20; OBI 1 = MO; HB, Be; and 2 = 4. 34, 28 + yt < 25 is the set of points interior to the circle t+ yt = 25. Tet the straight line x+y = I intersect the circle in A and B. Then all the points of the straight line segment AB, except A and B, are also interior to the circle. (C) is correet. 35. All the triangles so formed with A as one vertex are similar. Let hs be the side parallel to BC at adistance (1/8) AC fom Ay k= 1,3, *=%7 ae 108 (aa aC a aD ee Shy hy be thy = (10/8) (LE BH ET) = Part 3 36. If 2+ y= 4, then P = zy = 2(1 — 2) = —2¢ +2. The largest value of P occurs when x = 1/2 (see 1950, Problem 41). : P (maximum) = 1/4. 37, By similar triangles MN/z = 5/12, or MIN = 52/12. Also, NP = WC = 12-2 = 12-2 + 51/12 = (4 ~ 729/12, 38, 10/4 uv — (10u +f) = 9(¢ — w). Since both t and uw lie between 0 and 9, t — w cannot exceed 9. =. O(¢ — u) cannot exceed 81. The possible cubes are, therefore, 1, 8, 27, and 64, of which only 27 is divisible by 9 Mt — w) = 27; | |. t= w+ 3 so that the possibilities for N = 10t-+ u are 96, 85, 74, 63, 52, 41, 30, seven in all. wa 1 tn inch wll Th ht ed a wade gronty S224 Bho Oo 2 BEE em so that they meet midway between AF and. f= 95 and each man walks 36 miles, 18 THE CONTEST PROBLEM BOOK 40. When the vertex is on the z-axis, y = 0 and the roots of =# + bz — 8 = 0 are equal since this parabola touches the z-axis at only one point. «. — 32 = 0 and b = “LV/. 41, ‘There is no solution when a(—a) — (a ~ 1)(a + 1) = 0, that is, when a = £V2/2 (see 1950, Problem 14); solvefor z, obtaining z= (2a — 1)/(2at — 1), which is undefined when 2a — 1= 0, that is, when a = 3/2, 42, For integral values of n, #* equals either 4, —1, —i or +1. When fei, then i= Ii = —i/1, «.S canbe 0, -2, or +2, 43, On the line x = 4 the integral ordinates are 0, 1, 2, --+, 16; om the line z= 3 they are 0,1,2,--,9; on z=2 they are 0, 1,---, 4; on x= 1 they are 0, 1;and on 2 = 0 the integral ordinate is 0. + The number of lattice points is 17 + 10+ 54+2+1 = 35, 44. ABAD = 4CAB ~ XCAD = XCAB—~ 4CDA CAB ~ (4BAD + 4B). 24BAD = 4CAB ~ 4B = 30°; 2 MBAD = 15°, 45, Solving for x, we have = = y'/(y — 1), y x 1. Solving for y, we have y= (@t VE —G)/2% «x2 — 4) > 0, so that 2 >4 ‘oF z $0 and (A) is the answer. Note that choice (B) was quickly elimi- nated. ‘The values z = 4, y = 2 rule out choices (D) and (C). The values x= 5, y = (6 V/5)/2 rule out (E). 46, The center of the circle is the point of intersection of the perpendicular bisectors of the given chords. Draw the radius to the endpoint of one of the chords and use Pythagoras’ Theorem. em @ + (1/2)! = 65/4, r= V2, a= VB. \ 47. XY is the perpendicular bisector of AB. MB |. £ BMA is inceribed in a semi-circle and thus is right angle 2 BABM = 45°; AD = 90"; and AD ri. SOLUTIONS: 1958 EXAMINATION 129 48. Consider first the special ease where_M coincides with C. Then 4M = AC and BM + MC = BM = BC= AC. aM = BM + HC. es ‘To prove this in general, lay off HH (on MA) equal to MC. ACMA = 60%, AMCN in equilateral. We ean show that AACN & AMCB because AC = CB, CN = CH, and SACN = 4ACM — 60° = 4MCB.. BM = AN and 4M = N+ MN = BM + HC. 49. Let z and y_ be the two upper segments of the non-parallel sides Then 3+ yt2=94+6-yt4—z arty =8. Since aly = 4:6 = 2:3, 2y/3-ty = 8. y = 24/5, and = v) = 24/5:6/5 = 24:6 = 4:1, 50. A'ABB' is a trapezoid. Its median 00’ is perpendicular to AB. oo ee eee gee re (AW + BB) = 5 (AG + BG) = 5.48. +. Of is fixed distance from 0 on the perpendicular to AB, and therefore the point 0” is stationary. 1958 Solutions Part 1 zt a Fait sti t@-1) ze1_, #+1-@-0 zi 5, GEV VE ~ 2) + V3 2+ VIVE 032434) -2@)- 130 THE CONTEST PROBLEM BOOK 7. y= me +b is satisfied by (—1,1) and (3,9), 20 L=-m+b and 9 = 3m+b; b= 3; and the equation is y = 27 + 3. When y = 0, , inrational; YB = YF = gy itational; VDD = TTT = 2-10°9/TI, irrational; bility. 11. Replace V5=F = 1/5 =F by 5— 2 = x5 — 2). The solution set for this equation is |5, 1, —1} but the solution set for the original equation is (5, 1); rV5=2 If 5~ 20, divideby Vi 251; if 5-260, then x = 5. = and obtain ey np = 10E8 = log Plog (6/P) 12, log P = log s — log (1 + 4); n= SET DEE = eer oo ee eee 1 ey asaya ed 1 14, Set up the following table, a one-to-one correspondence between the elements of the boy set and the gil set: Boy number 1-2 3 ---; Girls danced with 5 6 7---b+ 4; gab +4 beg —4: by the formula for the last term of the arithmetic sequence 5, 6, 7, =~, 9: g= 54-1; bags 15, Take a special case, the square, where all the inseribed angles are equal; and each inscribed angle x = }(270"); 4x = 510°; SOLUTIONS: 1958 EXAMINATION 131 See 1958, Problem 28, 16, Let r be the radius of the eirele, and a, p, # the apothem, perimeter, and side of the regular hexagon. wt = 100r; r= 10; A = fap = 30s; a = A = 3-10-20/3 = 20073. 17. flog > log 2, log x > logs, 2 > 45 10; # = 2-10/73; log 2 > log 2Vz, x > 2V%, VE>2, 224. 18, A= wry 2A = alr ton); Dart = alot + Orn tt); r= nL + V2) the negative value 1 — /2 is not used. 19, at = or, BF = ca; r/e = a/b! = 1/9. (See 1956, Problem 38.) 82; (BY = 2, B= 5; Part 2 21, Consider the special case with A on C. The answer is obvious; A(QCED) = #5 A(QAOB) = Hp; ratio 2:1. 2. Wheny=6 2 =4 or -3; wheny=—-6 x=0or 1 Because of symmetry the particle may roll from (4, 6) to (1, —6) or from (~3, 6) to (0, ~6). In either ease the horizontal distance is 3. 2. w= 3; n= @ a= 3; mw— m= Har to 24. Let a = rate north = 30 mph, b = rate south = 120 mph, The average (see 1950, Problem 27). rl er the rod pis 2, 2a “ads 25, A= loge; a (og 2)(logs b) = AB = 30-120 30+ 199 ~ meh 2m 31 THE CONTEST PROBLEM BOOK = atate ta; (a; + 20) — 20+ 5(as + 20) — 20-4 --- 1 + 80 + Sap + 80+ +++ + Say + 80 = Blas + ast = + ay) + 800 = 5s + 80n; Sans Lie + 2-20 = Ou 3+3_ WD-3 yy aa ). From triangle ABC, 2+ n+ y + w= 180°. From tangle BED, m-+atwe-+0 = 1807 Nebytaeabbem Lybotee pore ee Lah ettt ia nen tte ternal 5+h- H : +a (e+ y) = 2 + dey + yf = at? + % = dlad + 2). Denote each side by a and the base by 26. 24 + 2 @ B= (a — Dat) = @—d)-16 caren = 8-6 = 48. 25s + 26¢ = 1000; 40 — 260/25 ‘The only solution in positive integers is ¢ = 25, 8 = M4. Let the roots be p,q where p = 295 p+ @ = 39 = —8/a; Oy" = By 1m = 24° = c/a, Division of 4 by 24" yields § = b/ac or 2H = Sac. SOLUTIONS: 1958 EXAMINATION 133 Bor+l> Ta e>h 0, 22-6 =0; (2-243) =05 2430; 2 Ir <0, #— 2-690; (r= e+2) = 0; r-380,26 (2+ lz 2) = 0; jz] +320; ela] jee = 40. When n= 1, af — aya = (—1)'; a = 10. When n= 2, a — am = (1); ay = 33. 18 THE CONTEST PROBLEM BOOK Py " 244). 2dC- BP 4 MO 42 aazn~ aan, 42-2, avai Baa +r: =F HU; 65 = aa +0), =2 V5. 44, Let p be the proposition “a is greater than b”. Let q be the proposition “cis greater than d’ Let 1 be the proposition “eis greater than J”. From the given, pq and ~q—>r. From the Rule of the Contrapositive, ~9 —» ~p and ~1— ‘Therefore, (A), (B), (C), (D) are not valid conclusions. 45, Let z= 100+ dollars y = 10+ c5nts ‘The face value of check is 1000a + 100b + 10¢ + d eonts. ‘The cashed value of check is 1000e + 100d + 10a + b cents ‘The difference is 990 + 98d — (990a + 994) = 1786. s. (1e + d) = (10a + ) = 18 or y= z= 18. Therefore, y can equal 22, fomee 1 7 se-1 +5 i} ‘The sum of a number and its reciprocal is numerically least when the number is +1. For r—1= 1, += 2 which isexcluded; ff—1= 1 and y= 1. Allother values of 2 in the interval given yield values of y less than —1; aia 2 = 59; dy _ (2x ~ 2)(22 ~ 2) ~ Ga" ~ 22 + 2912) a @r=2F 220, = 2 (excluded). By testing with values close to z = 0 to the right and to the left, we find that y is maximum when 2 = 0. SOLUTIONS: 1958 EXAMINATION 135 47. QPTR ~ AATQ; PR/AQ = PP/AP ea Pr = AT (4PAT = 4PBS = 4APT); PR iQ, PS = QF; PR + PS = 40+ OF ~ 4F; ASBP = 4 TPA = XTAP, A,Q,R,P_areconeyclic, ae PR = are AQ, PR = 4, PS ~ QF; PR+ RS = AP. 48, Let Q be the foot of the perpendicular from P to AB and let QB = 2. Then (PQ)' = x(10 — 2). P= VAO-DFC-H DP= VRO- HPO wy CP + DP = VHB + VIO Be. ‘This sum is greatest when /35— Bz = /I6F Bz, that is when = 5. Hence (E); fan ellipse through A and B with C and D as foci is the locus of points P’ such that CP’ + P'D = 10; since this ellipse lies entirely inside the given cirele (except at the points of tangency A and B), we see that CP + PD & 10 for all points P on the circle, the sign of equality holding only when P coincides with A or B. 199 9 tb +e" (at re (a + by + 10(a + bye + PS Hoe + 1Ola + bet + a” Since the expansion of (a + 5)" has m+ 1 distinct terms, in extended form the first term above has 11 distinet terms, the next has 10, ete. The total number of distinet terms is eee 0. DP = 2 =o; DP =y; PB/P'R = 1/4. y= DP +BP =1+BP = 1+ 4PB; PB = ya +4 =a) = 17 = 40, 2+ y= 17 ~ 3a, = 3 = 4 since for x = =5, for ie mm a aS a 4 since 3, y= 5, fe dye ym nde +b, y= de $17, B= IT. When x =a, 2+ y= 17 — 3a, 136 10, . The converse is: If a quadrilateral is a rectangle, then een: na dntindin tt. on THE CONTEST PROBLEM BOOK 1959 Solutions Part 1 S (old) = 62°; $ (new) = 61.52)" = 13.50s", Increase = 7.52* = 62"). Increase (%) = 126 Let 2 be the distance from P to AB. By similar triangles 1-2) 1 2-2 gr si-se ad (negative 29. root rejected). z . There are a variety of figures satisfying the given conditions, and not falling within the classifications (A), (B), (C), (D), for example, the tite”: 1yt 1 pt isthe 7B Oe = $68, 52 = 17 (256)"*(256) = (256) = (256)"" = 4 isa square, ‘The inverse is: Ifa quadrilateral is not square, then itis not a rectangle, Both statements are false; ‘use Venn diagrams to picture the sets mentioned. + (+d) = (0 + 20%; a/d = 3/1. 2 Gr + 13 = (¢ — 3)" + 4. The smallest value for this expression, 4,8 obtained when 2 = = 8d (a= —d is excluded); graph y = 2" — Gr + 3. The tuming point, whose ordinate is 4, is a ‘minimum point. (See 1950, Problem 41.) 140. Let the altitude from B to AC be A. Then since 1D altitude from D to AB is th. Let AB = x. Then fier 3 Bog = phe) 2 = 108 SOLUTIONS: 1959 EXAMINATION 137 11. Let loge 0625 = 2; 28 = 0625 = 4; zm —4, D+e ‘+e 12, Let a = the constant aad. 3. sum of numbers ‘umber of numbers * p= 1900 = 45 — 55 8 18, Arithmetic mean = + S = 50 X 38 = 1000; = 375, 14, Addition, subtraction, and multiplication with even integers always yield even integers. Note: This is « good opportunity to underscore operational restrictions ‘imposed by the available domain. 15,2 = 2aby at + Bm Qa; (@— BY = 0; and ab ‘Since the right triangle is isosceles, one of the acute angles is 45°. (z= 3)le 16. 1 1. 18. The sum of the fist. n positive integers is» = M+ 1) Since AM, = 5, the correct choice is (E). 18. Weg wet 1. singly 2. two at a time (same pan) 3, three at a time (eame pan) 4. two at a time (diff. pans) 5. three at a time (diff. pans) Ky -K®, one m2, 10 KO, wk 138 THE CONTEST PROBLEM BOOK aR 22, The median of a trapezoid goes through the midpoints of the diagonals. Let x be the length of the shorter base. * length of medi Meto =E43+5 OL 23, loge (a? ~ 18a) = 2; <1 = at — 15a; and «a= 20 or a= ~5, a te mre 100! 25, Let AB be a straight line segment with its midpoint at 3, and its right ‘end at B. Esch of the two intervals from A to 3 and from 3 to B is less than 4. Hence B is to the left of 7 and A is to the right of ~1; => 100 am if2<3, [3-2] <4 meas 3-2<4 « -1 3, |3—2/ <4 mems r-3<4, 42 <7, c e 2, a “%S Altitude CDF SOLUTIONS: 1959 EXAMINATION 139 27. The sum of the roots is. 1/é 28, The bisector of an angle of a triangle divides the opposite sides into segments proportional to the other two sides (see 1953, Problem 28) AM ogy Ch . MB @ 15 +4 (2 - 20 30, Let = B's time in seconds. Letting | represent the length ofthe track, 1 1 we have 3 (15) + £ (15) = 1. =, ee +e 50. 1 3 BL, Let 22 be the side of the emaller square. Then Be Fret s.r= V3. Let S be the side of the larger square. Then S=rV3=10; A= 100 32, The point A, the point of tangency, and the center of the circle, de- termine a right triangle with ove side 1, another side r, and the by- potenuse x +, where x is the shortest distance from A to the circle. (fy trees B + the terms of the H.P. are 3, 4, 6, 12 only (since 0 has no reciprocal). = = 25. We de+ 10; rem land rte C4 ate t tO and atte 35. Solving for z we have x(—m +n) = —mn + nt; 36. Let the triangle be ABC, with AB = 80, BC = a, CA 4B = 60". Let CD be the altitude to AB. Let 2 = BD. 40 THE CONTEST PROBLEM BOOK VE. 16st — r+ 1 4 does not satisfy the original equation); = 0 and y= 208, ce + VB — ¥, Thus = = 3, 5 (Rejent) xo) 2-2 Wrteeet eta = IZ) Bae O44 Ba++ +94; = 3 (04 90) = 450; S = 5, +S. 40, Let G be a point on BC s0 that FE = EG. Connect D_with @. ‘Then FDGB is. parallelogram. — :. D = 5, AF = 10, AB = 15. AL tte — P= e + 4) 42, Represent a,b, ¢ in terms of their prime factors. Then D is the product ‘of all the common prime factors, each factor taken as often as it appears the least number of times in a or b or c. Mf is the product of all the rnon-common prime factors, each factor taken as often as it appears the ‘greatest number of times in a or b oF ¢. ‘Therefore, MD may be less than abc, but it cannot exceed abe Obviously, MD equals abc when there are no common factors. 43, Leth be the altitude to side AB. Any other altitude ean be used equally well. Then right triangle ACD js similar to right triangle ECB because angle A and angle H are equal. Hence 27/25 = 40/h. To find h, we use the area: SOLUTIONS: 1959 EXAMINATION ui Area of ABC = 44-39 4 hm BAB 25-40 _ 125 op 30. 44, Let the roots be 1+ m and 14 with m and n both positive. Ltmtitn=—b and (1+ ml +2) =e. a=btetl=mn>0. log 2 = 1262, we have, with base 2, fogaa logy fog 2 2log3 = og 9) y= 8 46. rainy mornings _non-rainy mornings rainy = afternoons » ‘on-Fainy Fi : afternoons dmatbtete atbte=7, a=0,c+e=5, bte=6 Ae= 2, and a. 48, For k= 3 wecan have Iz', 12! +1, Lxt— 1, 2x', 82%, {The number / is known as the height of the polynomial. It is utilized in proving that the number of polynomials with integer coefcienta is a countable in- finity, the lowest one in the seal of infinities, 12 THE CONTEST PROBLEM BOOK ). Combine the terms in threes, to get the geometric series tty atetamto rearrange the terms into three series Vt bet) boi cr) -b en ). This problem may be interpreted as - ‘a miniature finite geometry of 4 lines and 6 points, so that each pair of lines has only 1 point in common, and 4 ‘each pair of points has only 1 line in common; P 4,2) = displayed as follows: | ‘Committees ‘Members | 1960 Solutions Part 1 Substituting 2 for x, wehave P+ h2+10=0 ah= ~9. For the six strokes there are five equal intervals so that the time interval between successive strokes is one second. For twelve strokes, then, eleven seconds are required. SOLUTIONS: 1960 EXAMINATION 3, 3. 40% of $10,000 is $4,000; 36% of $10,000 is $3,600; 4% of ($10,000 — $3,600) is $256. $3,600 + $256 ~ $3,856; «+ the difference is $4,000 — $3,856 ‘two successive discounts of 36% and 4% are equivalent to one discount of 38.56% (see 1950, Problem 22) 4. The triangle is equiangular with side 4. A= 85/4 = #Y3/4 = 45. 5. Substitute y# = 9 into 2+ y= 9 to obtain 2+ 9 = 9, so that 2 = 0, Therefore, the common solutions, and, hence, the intersection points are (0, 3) and (0, —3); + y= 9 represents a circle with radius 3 and centered at the origin. y = 9 represents the pair of lines y= +3 and y = —3, tangent to the circle at (0, 3) and (0, ~3). 6, Since 100 = md, d = 100/n. Let. # be the side of the inseribed square, d/J2 = 100/rv/2 = 50V2/r. : 7. Area II/Area = sRt/ar! = (2r)t/rt = Area II = 4 Areal = 4-4 = 16. 8 Let N = 2.52525 --, 100N = 252.52525---. Subtracting the first ‘equality from the second, we obtain 99N = 250; «NV = 250/99, Since this fraction is already in lowest terms, we have 250 + 99 = 349, EHV P +b (at =e (tb +atb—-o ator” @tethete-v atbme . wit with (a+ o)* Ut 10, To negate the given statement we say: It is false that all men are good drivers, that is, atleast one man is a bad driver 11, The product of the roots, here, is equal to 2k — 1 2M ola, Bad ‘The discriminant = (—3k)? — 4-1-@H — 1) = B44 = ‘the roots are irrational 5 therefore 12, The locus is @ circle, centered at the given point, having a radius equal to that of the given cirles m4 13, 14 THE CONTEST PROBLEM BOOK ‘The given lines interect pairwise in the three points (0,2), B(4/3, ~ 2) and_C(=4/3, ~2). ‘These three points are the vertices of triangle with 4@ = AB,’ each of length 4/10/3, and BC = 8/3, £0 that (B) isthe correct choioe. Bei Stan bet, be — be bea. - 4a, = -a/-b if 16. Since the triangles are similar, we have A/a = P/p = R/r = /R/VE sith ce 16 t= 29-4854 41 = 2 (ht Bn ne ae, rat et, es et Pon) 17. We have N = 8-108-2-*” with N = 800; solve for 2: 800 = 8-10%-2*”, T= 102%, 2? = 10%, 2! = 10, 2 = 104 18, 3 B18 tH BIE ws ey “ +h 2 — & and the correct choice is (E). iD. For (A) we have n+ n+1+n+2 = 3n+3 = 46, Foe) webwe +4 24m 42 Serr O eae ‘imply inn» Fe Onetne ntntitatoennne deed silat ete For (D) we have 2n + 2n + 2+ 2n +44 2n+ 6 Viet =, weet nen» For @\nebne tLe mea OEE went i8lo sing i x wn (é- + 4 878 Gann gye BF cyte 4 SES Gotan + 876 TEs (2) —4)"; the coeficien 4; wa OCB, BY" 4)" = {he required coeiient in pe SES (a = 1

You might also like